Vous êtes sur la page 1sur 89

British

Astronomy and Astrophysics Olympiad


2015 -2017

PROBLEMS

Solutions

Compiled By
Science Olympiad Blog
British Astronomy and Astrophysics Olympiad 2016-2017
Astronomy & Astrophysics Competition Paper

Monday 23rd January 2017


Instructions
Time: 3 hours plus 15 minutes reading time (no writing permitted). Approx 35 minutes per question.

Questions: All five questions should be attempted.

Marks: The questions carry similar marks.

Solutions: Answers and calculations are to be written on loose paper or in examination booklets.
Students should ensure their name and school is clearly written on all answer sheets and pages are
numbered. A standard formula booklet with standard physical constants should be supplied.

Instructions: To accommodate students sitting the paper at different times, please do not discuss any
aspect of the paper on the internet until 8 am Saturday 28th January.

Clarity: Solutions must be written legibly, in black pen (the papers are photocopied), and working
down the page. Scribble will not be marked and overall clarity is an important aspect of this exam
paper.

Eligibility: The International Olympiad will be held during November 2017; all sixth form students are
eligible to participate, even if they will be attending university in November.

Training Dates and the International Astronomy and Astrophysics Olympiad (IOAA)

The IOAA this year will be held in Phuket, Thailand, from 12th to 21st November 2017.

The team will be selected from sixth form students taking this paper and Y12 students taking the AS
Challenge in March. The best students eligible to represent the UK at the IOAA will be invited to attend
the Training Camp to be held in the Physics Department at the University of Oxford, (Tuesday 4th
April to Friday 7th April 2017). Astronomy material will be covered; problem solving skills and
observational skills (telescope and naked eye observations) will be developed. At the Training Camp a
data analysis exam and a short theory paper will be sat. Five students (plus one reserve) will be
selected for further training. From May there will be mentoring by email to cover some topics and
problems, followed by a training camp in the summer and also one in the autumn.
Important Constants

Constant Symbol Value


Speed of light c 3.00 108 m s1
Earths rotation period 1 day 24 hours
Earths orbital period 1 year 365.25 days
parsec pc 3.09 1016 m
Astronomical Unit AU 1.49 1011 m
Radius of the Earth R 6.37 106 m
Semi-major axis of the Earths orbit 1 AU
Radius of the Sun R 6.96 108 m
Mass of the Sun M 1.99 1030 kg
Mass of the Earth M 5.97 1024 kg
Luminosity of the Sun L 3.85 1026 W
Gravitational constant G 6.67 1011 m3 kg1 s2

You might find the diagram of an elliptical orbit below useful in solving some of the questions:

Elements of an elliptic orbit: a = OA (= OP) semi-major axis


a=q OB (= OC) semi-minor axis
b2
e= 1 a2
eccentricity
F focus
P periapsis (point nearest to F)
A apoapsis (point furthest from F)

Keplers Third Law: For an elliptical orbit, the square of the period, T , of orbit of an object about the
focus is proportional to the cube of the semi-major axis, a (the average of the minimum and maximum
distances from the Sun). The constant of proportionality is 4 2 /GM , where M is the mass of the
central object and G is the universal gravitational constant.

Magnitudes: The apparent magnitudes of two objects, m1 and m0 , are related to their apparent
brightnesses, b1 and b0 , via the formula:
b1
= 100.4(m1 m0 )
b0
British Physics Olympiad Sponsors

Worshipful Company of Scientific


Instrument Makers
Qu 1. Martian GPS
On Earth the Global Positioning System (GPS) requires a minimum of 24 satellites in orbit at any one
time (there are typically more than that to allow for redundancies, with the current constellation having
more than 30) so that at least 4 are visible above the horizon from anywhere on Earth (necessary for an
x, y, z and time co-ordinate). This is achieved by having 6 different orbital planes, separated by 60 , and
each orbital plane has 4 satellites.

Figure 1: The current set up of the GPS system used on Earth.


Credits: Left: Peter H. Dana, University of Colorado;
Right: GPS Standard Positioning Service Specification, 4th edition

The orbits are essentially circular with an eccentricity < 0.02, an inclination of 55 , and an orbital period
of exactly half a sidereal day (called a semi-synchronous orbit). The receiving angle of each satellites
antenna needs to be about 27.8 , and hence about 38% of the Earths surface is within each satellites
footprint (see Figure 1), allowing the excellent coverage required.

a. Given that the Earths sidereal day is 23h 56 mins, calculate the orbital radius of a GPS satellite.
Express your answer in units of R .

b. How long would it take a radio signal to travel directly between a satellite and its closest neighbour
in its orbital plane (assuming theyre evenly spaced)? How far would a car on a motorway (with
a speed of 30 m s1 ) travel in that time? [This can be taken to be a very crude estimate of the
positional accuracy of the system for that car.]

In the future we hope to colonise Mars, and so for navigation purposes it is likely that a type of GPS
system will eventually be established on Mars too. Mars has a mass of 6.42 1023 kg, a mean radius of
3390 km, a sidereal day of 24h 37 mins, and two (low mass) moons with essentially circular orbits and
semi-major axes of 9377 km (Phobos) and 23 460 km (Deimos).

c. Using suitable calculations, explore the viability of a 24-satellite GPS constellation similar to the
one used on Earth, in a semi-synchronous Martian orbit, by considering:

(i) Would the moons prevent such an orbit?


(ii) How would the GPS positional accuracy compare to Earth?
(iii) What would the receiving angle of each satellites antenna need to be, and what would be
the associated satellite footprint? By comparing these with the ones utilised by Earths GPS,
make a final comment on the viability of future Martian GPS.

1
Qu 2. Hohmann Transfer Orbits
In order to move a spacecraft between orbits we must apply a thrust using rockets, which changes the
velocity of the spacecraft by v. In this question we will ignore changes in the mass of the spacecraft
due to the burning of fuel.

q an object with mass M (where m  M )


For an object of mass m in a circular orbit of radius r around
GM
the orbital velocity, vorb , is given by the formula vorb = r .

a. Show that vorb in low Earth orbit (LEO; about 200 km above the surface), is about 8 km s1 . This
is an estimate of the v the rockets need to provide for the spacecraft to reach LEO.

An economical route to take when travelling between planets is called a Hohmann transfer orbit. This
is an ellipse for which the perihelion coincides with the inner planetary orbit (with radius rA ) and the
aphelion coincides with the outer planetary orbit (with radius rB ). It is achieved by increasing the
velocity of the spacecraft at point A by vA before then increasing it again at point B by vB .

Figure 2: A diagram of a Hohmann transfer orbit between an inner and outer planet

For an ellipse with semi-major axis a it can be shown that the velocity v, at a distance r from mass M ,
can be written as:  
2 2 1
v = GM
r a

b. Derive expressions for vA and vB by comparing their circular orbital speeds with their transfer
orbit speeds. Simplify your final expressions to include G, M , rA and rB only.

c. Approximating Mars orbit as circular with a radius of 1.52 AU, calculate the v to go from Earth
LEO to Mars i.e. v = |vA | + |vB |. Compare your answer to the v to reach Earth LEO.

d. Derive an expression for the total time spent on the transfer orbit, tH , and calculate it for an Earth
to Mars transfer. Give your answer in months. (Use 1 month = 30 days.)

e. Hence calculate the direct distance between Earth and Mars at the moment the spacecraft reaches
Mars. How long would it take a radio message from the spacecraft to reach Earth?

f. How long would any astronauts on board the spacecraft need to wait until they could use a
Hohmann transfer orbit to return to Earth? Hence calculate the total duration of the mission.

2
Qu 3. Starkiller Base
As part of their plan to rule the galaxy the First Order has created the Starkiller Base. Built within an ice
planet and with a superweapon capable of destroying entire star systems, it is charged using the power of
stars. The Starkiller Base has moved into the solar system and seeks to use the Sun to power its weapon
to destroy the Earth.

Figure 3: The Starkiller Base charging its superweapon by draining energy from the local star.
Credit: Star Wars: The Force Awakens, Lucasfilm.

For this question you will need that the gravitational binding energy, U , of a uniform density spherical
object with mass M and radius R is given by

3GM 2
U=
5R
and that the mass-luminosity relation of low-mass main sequence stars is given by L M 4 .

a. Assume the Sun was initially made of pure hydrogen, carries out nuclear fusion at a constant rate
and will continue to do so until the hydrogen in its core is used up. If the mass of the core is 10%
of the mass of the star, and 0.7% of the mass in each fusion reaction is converted into energy, show
that the Suns lifespan on the main sequence is approximately 10 billion years.

b. The Starkiller Base is able to stop nuclear fusion in the Suns core.

(i) At its current luminosity, how long would it take the Sun to radiate away all of its
gravitational binding energy? (This is an estimate of how long it would take to drain a
whole star when radiatively charging the superweapon.)
(ii) How does your value compare to the main sequence lifetime of the Sun calculated in part a.?
(iii) Comment on whether there were (or will be) any events in the life of the Sun with a timescale
of this order of magnitude.

c. In practice, the gravitational binding energy of the Earth is much lower than that of the Sun, and so
the First Order would not need to drain the whole star to get enough energy to destroy the Earth.
Assuming the weapon is able to channel towards it all the energy being radiated from the Suns
entire surface, how long would it take them to charge the superweapon sufficiently to do this?

3
The First Order find that radiative charging of the weapon is too slow to satisfy their plans for galactic
domination, and so instead the weapon charging process compresses and stores part of the star within the
Starkiller Base (as shown in Figure 3). To avoid creating a black hole, the First Order cannot compress
stellar matter below its Schwarzschild radius, RS = 2GM/c2 .

d. Taking the Starkiller Bases ice planet to have a diameter of 660 km, show that the Sun can be
safely contained, even if it was fully drained.

e. The Starkiller Base wants to destroy all the planets in a stellar system on the far side of the galaxy
and so drains 0.10 M from the Sun to charge its weapon. Assuming that the U per unit volume
of the Sun stays approximately constant during this process, calculate:

(i) The new luminosity of the Sun.


(ii) The new radius of the Sun.
(iii) The new temperature of the surface of the Sun (current T = 5780 K), and suggest (with a
suitable calculation) what change will be seen in terms of its colour.

The Resistance defeat the First Order and destroy the Starkiller Base when it was almost fully charged.
Upon releasing the energy stored in the base it causes the planet to turn into a small star.

Figure 4: The Resistance fighters escaping the Starkiller Base as it turns into a star.
Credit: Star Wars: The Force Awakens, Lucasfilm.

f. Assume that at the moment of destruction of the Starkiller Base the mass of the new star formed
is equal to the mass drained from the Sun (0.10 M ). Derive an expression for the main sequence
lifetime in terms of stellar mass, and hence calculate the main sequence lifetime of this new star.

4
Qu 4. Hannys Voorwerp
Hannys Voorwerp (Dutch for object) is a rare type of astronomical object discovered in 2007 by the
school teacher Hanny van Arkel whilst participating as a volunteer in the Galaxy Zoo project. When
inspecting the image of the galaxy IC 2497 in the constellation Leo Minor, she observed a bright green
blob close to the galaxy.

Figure 5: HST image of galaxy IC 2497 and the glowing Voorwerp below it.
Credit: Keel et al. (2012) & Galaxy Zoo.

Subsequent observations have shown that the galaxy IC 2497 is at a redshift of z = 0.05, with the
Voorwerp at a similar distance and with a projected angular separation of 20 arcseconds from the centre
of the galaxy (3600 arcseconds = 1 ). Radio observations suggest that the Voorwerp is a massive cloud
of gas, made of ionized hydrogen, with a size of 10 kpc and a mass of 1011 M . It is probably a cloud
of gas that was stripped from the galaxy during a merger with another nearby galaxy.

In this question you will explore the cause of the glow of the Voorwerp and will learn about a new
type of an astronomical object; a quasar.

a. Given that Hubbles constant is measured as H0 = 70 km s1 Mpc1 , calculate the distance to


the galaxy (in Mpc).

The rate of ionizing photons from a source (in photons per second) can be expressed as:

S = V n2

where V is the volume of the ionized region, n is the number density of the ionized gas and is the
ionization coefficient, = 2.6 1013 cm3 s1 .

b. Calculate the power (luminosity) of the source required to completely ionize the Voorwerp
(assumed to be spherical), given that the mass of a hydrogen atom is 1.67 1027 kg and the
ionization energy of hydrogen is 13.6 eV, where 1 eV = 1.60 1019 J.

5
One possible source of ionizing radiation is the jet arising from the accretion of material onto the
supermassive black hole (SMBH) situated in the centre of the galaxy. This produces an enormous
amount of energy, greatly brightening the galaxy; a galaxy shining due to this process is known as a
quasar.

c. The gravitational potential energy of the material falling to radius R, which in this case is a black
hole with radius equal to the Schwarzschild radius, RS = 2GM/c2 , at a mass accretion rate
m m/t, is converted into radiation with an efficiency of . Show that the power (luminosity)
output of the SMBH is given by:
1
L = mc2 .
2
d. The typical mass accretion rate onto an active SMBH is 2 M yr1 and the typical efficiency is
= 0.1. Calculate the typical luminosity of a quasar. Compare the luminosity of the quasar with
the power needed to ionize the Voorwerp.

Detailed astronomical observations have shown than the nucleus of the galaxy has a modest luminosity
of L < 1033 W, thus the black hole in IC 2497 is not currently active (i.e. the accretion rate is very
low). Quasars are thought to ignite every time the black hole starts accreting a fresh source of matter,
and switch off once that supply is exhausted. Therefore, this might be the first evidence of a quasar
switching off recently (by astronomical standards), with the Voorwerp reflecting the light emitted by the
quasar whilst it was still active. This would make the Voorwerp a quasar ionization echo and IC 2497
the nearest galaxy to us to host a quasar.

e. Calculate the projected physical separation, rp , between the galaxy and the Voorwerp.

f. Derive an expression for the difference in the light travel time between photons travelling directly
to Earth from the galaxy and photons reflected off the Voorwerp first. Give your formula as a
function of rp and , where is the angle between the lines of sight to the Earth and to the centre
of the Voorwerp as measured by an observer at the centre of IC 2497. (For example = 90
would correspond to the galaxy and Voorwerp both being the exact same distance from the Earth,
and so the projected distance rp is therefore also the true distance between them.)

g. High precision measurements showed that the Voorwerp is slightly further away than the galaxy,
and so = 125 . Use this with your expression from the previous part of the question to estimate
an upper limit for the number of years that have passed since the quasar was last active.

6
Qu 5. Imaging an Exoplanet
Recently a group of researchers announced that they had discovered an Earth-sized exoplanet around
our nearest star, Proxima Centauri. Its closeness raises an intriguing possibility about whether or not we
might be able to image it directly using telescopes. The difficulty comes from the small angular scales
that need to be resolved and the extreme differences in brightness between the reflected light from the
planet and the light given out by the star.

Figure 6: Artists impression of the view from the surface of Proxima Centauri b.
Credit: ESO / M. Kornmesser

Data about the star and the planet are summarised below:

Proxima Centauri (star) Proxima Centauri b (planet)


Distance 1.295 pc Orbital period 11.186 days
Mass 0.123 M Mass (min) 1.27 M
Radius 0.141 R Radius (min) 1.1 R
Surface temperature 3042 K
Apparent magnitude 11.13

The following formulae may also be helpful:


   
d L
m M = 5 log M M = 2.5 log m = 2.5 log CR
10 L
where m is the apparent magnitude, M is the absolute magnitude, d is the distance in parsecs, and the
contrast ratio (CR) is defined as the ratio of fluxes from the star and planet, CR = ffplanet
star
.

a. Calculate the maximum angular separation between the star and the planet, assuming a circular
orbit. Give your answer in arcseconds (3600 arcseconds = 1 ).

b. Determine the luminosity of the star and hence calculate the flux received on the Earth (in W m2 )
from both the star and the planet. Use them to work out the contrast ratio and thus the apparent
magnitude of the planet. Assume the planet reflects half of the incident light and that M = 4.83.

7
The resolving power of a diffraction limited telescope is given by


min = 1.22
D
where is the wavelength being observed at, D is the diameter of the telescope aperture, and min is
the smallest angular separation (in radians) the telescope can distinguish.

Data about some current and planned telescopes are summarised below:

Telescope Diameter (m) Faintest m detectable


Hubble Space Telescope (HST) 2.4 31
Keck II (based in Hawaii) 10.0 (variable)
James Webb Space Telescope (JWST) 6.5 34

c. Verify that the HST (which is diffraction limited since its in space) would be sensitive enough to
image the planet in the visible, but is unable to resolve it from its host star (take = 550 nm).

Ground-based telescopes have bigger mirrors than the HST, but are not diffraction limited due to
movements in the atmosphere and so need to be fitted with adaptive optics (AO) to compensate for
this effect. However, even with perfect AO the faintest object the telescope can detect is limited by the
brightness of the atmosphere.

The signal-to-noise ratio (SN R) can be approximated as:

f At
SN R
f At + bt
where f is the flux from the object (in photons m2 s1 ), A is the area of the telescope mirror,  is the
overall efficiency of the telescope and detector, b is the flux from the sky (in photons s1 ), and t is the
length of the exposure.

d. Calculate the exposure time needed for a Keck II image of the exoplanet to have an SN R of 3
(i.e. barely detectable). Assume that the telescope has perfect AO (so it is diffraction limited),
is observed at the longest wavelength for which the planet can still be resolved from the star, all
the received flux from the planet consists of photons of that longest wavelength,  = 0.1 and
b = 109 photons s1 (so b  f ). Comment on your answer.

The James Webb Space Telescope (JWST) is the successor to the HST and is due to launch in 2018.
It should be able to both resolve the system and cope with the contrast ratio. Since it is in space it is
diffraction limited, and the SN R should be dominated by the flux from the planet (i.e. f  b).

e. How long an exposure would JWST need in order to get the same SN R as Keck II, again if
observed at the longest wavelength for which the planet can still be resolved from the star by the
telescope? (Make similar assumptions about the received flux and use the same value of .)

END OF PAPER

Questions proposed by:


Dr Alex Calverley (Bedford School)
Dr Emile Doran (The Langley Academy)
Sandor Kruk (University of Oxford)

8
BAAO 2016/17 Solutions and Marking Guidelines

Note for markers:

Answers to two or three significant figures are generally acceptable. The solution may give more in
order to make the calculation clear.
There are multiple ways to solve some of the questions; please accept all good solutions that arrive
at the correct answer. If a candidate gets the final (numerical) answer then allow them all the marks
for that part of the question (as indicated in red), so long as there are no unphysical / nonsensical
steps or assumptions made.

Q1 - Martian GPS [Total = 20]

a. Given that the Earths sidereal day is 23h 56 mins, calculate the orbital radius of a GPS satellite.
Express your answer in units of R.

[1]

[1]

[1] [3]

b. How long would it take a radio signal to travel directly between a satellite and its closest neighbour
in its orbital plane (assuming theyre evenly spaced)? How far would a car on a motorway (with a
speed of 30 m s-1) travel in that time? [This can be taken to be a very crude estimate of the positional
accuracy of the system for that car.].

Closest neighbour in orbital plane should be 90 away (since evenly spaced) so can use Pythagoras
[1]

Time for the signal to travel that distance:


[1] [2]

Distance travelled by a car on a motorway in that time:


[1] [1]

[In practice the positional accuracy of a GPS system is much harder to calculate; at low speeds it is
typically a function of fluctuations and reflections of the signal within the atmosphere, as well as the
presence of objects that might block the signal]
c. Using suitable calculations, explore the viability of a 24-satellite GPS constellation similar to the one
used on Earth, in a semi-synchronous Martian orbit, by considering:
i. Would the moons prevent such an orbit?

[1]

[1]
[1] [3]
This is about the distance to Diemos and about 3500 km (about RM) away from Phobos so
the moons should not provide a problem for any GPS satellite constellation [1] [1]

ii. How would the GPS positional accuracy compare to Earth?

(Using similar reasoning to part b.)


[1]
(or ) [1] [2]
The positional accuracy is about twice as good as on Earth [1] [1]

iii. What would the receiving angle of each satellites antenna need to be, and what would be
the associated satellite footprint? By comparing these with the ones utilised by Earths GPS,
make a final comment on the viability of future Martian GPS.

Receiving angle:
[1]
[1] [2]

The area of a 'zone' of a sphere is 2Rh where h is the radial


height of the zone. From the geometry of the situation:
[1]
[1]

Fraction of surface area:

[1] [3]

Receiving angle similar to Earth's so can use current GPS satellite technology [0.5]
Satellite footprint similar to Earth's so should get sufficient coverage [0.5]
Martian GPS system is viable [1] [2]
Q2 - Hohmann Transfer [Total = 20]

a. Show that vorb in low Earth orbit (LEO; about 200 km above the surface) is about 8 km s-1. This is an
estimate of the v the rockets need to provide for the spacecraft to reach LEO.


[1]

[1] [2]

[In practice a v of 8 km s-1 assumes no external forces, but atmospheric drag can increase the
necessary v by 1.3 1.8 km s-1. When travelling between objects in space, however, such drag
forces are absent and so the v calculated is much more accurate]

b. Derive expressions for vA and vB by comparing their circular orbital speeds with their transfer orbit
speeds. Simplify your final expressions to include G, M, rA and rB only.

[1]

But 2a = rA + rB
[1] [2]
Similarly:
[1] [1]

[These equations have been written so that the change in speed is positive, however give full credit
for reversed signs (so long as they are consistent)]

c. Approximating Mars' orbit as circular with a radius of 1.52 AU, calculate the v to go from Earth LEO
to Mars i.e. v = |vA|+|vB|. Compare your answer to the v to reach Earth LEO.

[1]

[1]

[1] [3]

This is less than the v to get into LEO [1] [1]


(So most of the effort needed in going to Mars simply comes from leaving Earth)
[The v calculated here would be for the spacecraft to enter a circular orbit around the Sun at the
same distance as Mars, but this would not constitute landing the extra v to get to the Martian
surface increases the total for the whole transfer to roughly 8 km s-1 (same as to get into LEO). Other,
more complicated routes can be taken, some of which offer substantial efficiencies so the v for the
trip can be much lower (although the time taken to complete the manoeuvre will be longer, and the
v to reach LEO will still be the biggest single step)]

d. Derive an expression for the total time spent on the transfer orbit, tH, and calculate it for an Earth to
Mars transfer. Give your answer in months. (Use 1 month = 30 days).

From Kepler's third law:


Since the spacecraft only covers half of the ellipse the time on the journey is half the period, and
given that 2a = rA + rB then:
[1] [1]

[1]

[1] [2]

e. Hence calculate the direct distance between Earth and Mars at the moment the spacecraft reaches
Mars. How long would it take a radio message from the spacecraft to reach Earth?

x
Initially Earth is at A. When the spacecraft reaches B (after 8.56
A'
months), the Earth has moved round the Sun in its orbit and is
now at A'.

Angle between A' and B:


[1]

Using the cosine rule:

[1] [2]
Since radio waves travel at the speed of light the time taken by the message is:
[1] [1]

[In practice, due to the eccentricity of Mars' orbit, the signal transmission time varies depending on
the year the spacecraft was launched. When the rover Curiosity arrived at Mars the engineers
described the landing as 7 minutes of terror, since the signal from the spacecraft would take 14
minutes to reach Earth but the time to transverse the Martian atmosphere was only 7 minutes
(hence the process had to be completely automated)]
f. How long would any astronauts on board the spacecraft need to wait until they could use a Hohmann
transfer orbit to return to Earth? Hence calculate the total duration of the mission.

During the transfer the Earth moves by , so the


A" spacecraft should launch from Mars when Earth is at position A"
(exhibiting symmetry with when it arrived).

Since the planets move anti-clockwise in this diagram the angle


covered by Earth from A' to A" is [1]

From Kepler's third law, the period of Mars [1]

Therefore, the relative angular velocity of Earth if Mars' motion is subtracted out is:
[1]
-1 -8 -1
(allow any equivalent units e.g. 168 year , 9.38 10 rad s etc.)

Consequently, the time the astronauts need to wait for Earth to get from position A' to A" is:
[1] [4]

Thus the total duration of a return mission to Mars is:


[1] [1]

[Shorter missions are possible, but would require a greater v and hence need much more fuel - any
future mission will have to balance the cost (and mass) of more fuel on a fast trip with the cost (and
mass) of more supplies on a slow trip]
Q3 - Starkiller Base [Total = 20]

a. Assume the Sun was initially made of pure hydrogen, carries out nuclear fusion at a constant rate
and will continue to do so until the hydrogen in its core is used up. If the mass of the core is 10% of
the star, and 0.7% of the mass in each fusion reaction is converted into energy, show that the Suns
lifespan on the main sequence is approximately 10 billion years.

Time on main sequence = total nuclear energy available / luminosity

[1]
[1] [2]

b. The Starkiller Base is able to stop nuclear fusion in the Suns core
i. At its current luminosity, how long would it take the Sun to radiate away all of its
gravitational binding energy? (This is an estimate of how long it would take to drain a whole
star when radiatively charging the superweapon.)

Time radiating energy = total gravitational binding energy / luminosity

[1]
[1] [2]

ii. How does your value compare to the main sequence lifetime of the Sun calculated in part a.?

This is much shorter than tMS [1] [1]

iii. Comment on whether there were (or will be) any events in the life of the Sun with a timescale
of this order of magnitude.

An event in the Sun's life that happened on a timescale of this order of magnitude is the
gravitational collapse of the protostar before it joined the main sequence [1] [1]

[The Sun will also be on the asymptotic giant branch (AGB) for a similar order of magnitude
of time - this is when the core is completely carbon / oxygen (but no longer undergoing
fusion) and there is a spherical shell of helium burning happening just outside the core (with
a shell outside that of hydrogen burning). Credit this answer too if a student mentions it.]

c. In practice, the gravitational binding energy of the Earth is much lower than that of the Sun, and so
the First Order would not need to drain the whole star to get enough energy to destroy the Earth.
Assuming the weapon is able to channel towards it all the energy being radiated from the Suns
entire surface, how long would it take them to charge the superweapon sufficiently to do this?

Time charging the weapon = total energy needed / rate of energy transfer

[1]

[1] [2]
(So it would only take a week to absorb enough energy from the Sun to destroy the Earth!)
d. Taking the Starkiller Bases ice planet to have a diameter of 660 km, show that the Sun can be safely
contained, even if it was fully drained.

Need to work out the Schwarzschild radius for the Sun, and compare it to the size of the base
[1] [1]

(About 100 times) Smaller than the radius of the base the Sun can be safely contained [1] [1]

e. The Starkiller Base wants to destroy all the planets in a stellar system on the far side of the galaxy
and so drains 0.10 M from the Sun to charge its weapon. Assuming that the U per unit volume of
the Sun stays approximately constant during this process, calculate:
i. The new luminosity of the Sun.

Need to use the mass-luminosity relation for main sequence stars (L M4) to work out the
luminosity of a 0.9 M star (since that is the new mass of the Sun)


[1] [1]

ii. The new radius of the Sun.

Energy density = U / V = constant

[1]

[1] [2]

iii. The new temperature of the surface of the Sun (current T = 5780 K), and suggest (with a
suitable calculation) what change will be seen in terms of its colour.

Need to use the Stephan-Boltzmann Law to get the new temperature and then Wien's Law
to determine the effect on the peak wavelength (and hence the colour)

[1]

[1] [2]
[1] [1]
This is a longer wavelength than the current peak (500 nm) so the Sun is redder [1] [1]
f. Assume that at the moment of destruction of the Starkiller Base the mass of the new star formed is
equal to the mass drained from the Sun (0.10 M). Derive an expression for the main sequence
lifetime in terms of stellar mass, and hence calculate the main sequence lifetime of this new star.

We can combine the mass-luminosity relation with the expression we used in part a.
[1] [1]

[1]
[1] [2]
Q4 - Hanny's Voorwerp [Total = 20]

a. Given that Hubbles constant is measured as H0 = 70 km s-1 Mpc-1, calculate the distance to the galaxy
(in Mpc).

Need to turn the redshift into a recessional velocity and then combine with Hubble's Law
[1]
[1] [2]

b. Calculate the power (luminosity) of the source required to completely ionize the Voorwerp (assumed
to be spherical), given that the mass of a hydrogen atom is 1.67 10-27 kg and the ionization energy
of hydrogen is 13.6 eV, where 1 eV = 1.60 10-19 J.

Given we know the radius of the cloud (10 kpc) and the mass (1011 M) we can work out the number
density of hydrogen atoms

[1]

[1]

[1]
(watch that the units of are converted correctly to SI)

The luminosity can then be calculated as we know the energy of each photon
[1]
[1] [5]

(Allow full credit for interpreting the 10 kpc 'size' of the cloud to mean its diameter rather than its
radius, giving S* = 2.39 1056 photons s-1 and L = 5.20 1038 W)
[Working out S* directly may prove difficult for some calculators as (M/mH)2 may exceed their largest
power of ten, in which case students should work out S* and then square it later.]

c. The gravitational potential energy of the material falling to radius R, which in this case is a black hole
with radius equal to the Schwarzschild radius, RS = 2GM/c2, at a mass accretion rate , is
converted into radiation with an efficiency of . Show that the power (luminosity) output of the SMBH
is given by .

We know the gravitational potential energy of a particle of mass m at the Schwarzschild radius is the
same as the kinetic energy it has gained moving from infinity to that point, so
[1]

Given that a fraction is converted into radiation and the given mass accretion rate then
[1] [2]
(So the maximum energy you can get from a black hole is half the rest mass energy of the material
falling in this is a much more efficient process for generating energy than the 0.7% you get from
nuclear fusion in stars, which in themselves are much more efficient than chemical reactions!)

d. The typical mass accretion rate onto an active SMBH is 2 M yr-1 and the typical efficiency is =
0.1. Calculate the typical luminosity of a quasar. Compare the luminosity of the quasar with the
power needed to ionize the Voorwerp.

Need to convert the mass accretion rate into kg s-1 and then put into the formula
[1]
[1] [2]

The luminosity of the quasar is high enough to ionize the Voorwerp [1] [1]

e. Calculate the projected physical separation, rp, between the galaxy and the Voorwerp.

Since the angle is so small, we can use the small angle approximation for tan
[0.5]
[0.5]
[1] [2]

f. Derive an expression for the difference in the light travel time between photons travelling directly to
Earth from the galaxy and photons reflected off the Voorwerp first. Give your formula as a function
of rp and , where is the angle between the lines of sight to the Earth and to the centre of the
Voorwerp as measured by an observer at the centre of IC 2497. (For example = 90 would
correspond to the galaxy and Voorwerp both being the exact same distance from the Earth, and so
the projected distance rp is therefore also the true distance between them.)

Given the small angular separation we can treat the light


rays from the galaxy to Earth and from the Voorwerp to
Earth as essentially parallel, and so the difference in light
travel time comes from the extra distance travelled in being
reflected off the Voorwerp

Relevant diagram, suitably labelled [2]


Extra distance = x + y
[1]
[1]
[1] [5]
(alternative form)

[Allow any equivalent formula, for example expressing it in terms of csc and cot , so long as some
attempt has been made to simplify it. It is quicker and simpler to derive if is assumed to be acute
we show it this way in case students see that the angle is obtuse from the next part of the question
and want to have a consistent picture throughout]
g. High precision measurements showed that the Voorwerp is slightly further away than the galaxy, and
so = 125. Use this with your expression from the previous part of the question to estimate an upper
limit for the number of years that have passed since the quasar was last active.

[0.5]
[0.5] [1]

(This is remarkably recent on astronomical timescales!)


Q5 - Imaging an Exoplanet [Total = 20]

a. Calculate the maximum angular separation between the star and the planet, assuming a circular
orbit. Give your answer in arcseconds (where 3600 arcseconds = 1).

[1]
[1]
[0.5]
[0.5] [3]

b. Determine the luminosity of the star and hence calculate the flux received on the Earth (in W m-2)
from both the star and the planet. Use them to work out the contrast ratio and thus the apparent
magnitude of the planet. Assume the planet reflects half of the incident light and that = 4.83.

[1]

[1] [2]

[1] [1]

[1]

[1] [2]

[1] [1]
[1]
[1] [2]

[Since we only know the minimum radius of the exoplanet it could be larger and hence brighter,
however it may also reflect less than half the incident light from the star and so be fainter in
practice the numbers used here are an optimistic estimate and it is more likely to be fainter.]
(Accepted alternative methods: if they use Stephan-Boltzmann's Law then L = 5.88 1023 W, fstar =
2.92 10-11 W m-2, and fplanet = 3.39 10-18 W m-2, though the contrast ratio and magnitude should be
the same. Also accept if they assume only the day side is able to reflect and hence the apparent
magnitude brightens to 27.9)
c. Verify that the HST (which is diffraction limited since it's in space) would be sensitive enough to
image the planet in the visible, but is unable to resolve it from its host star (take = 550 nm).

[0.5]
Since HST > max then the HST can't resolve it [0.5] [1]
Apparent magnitude of planet (28.5) is brighter (greater) than limiting magnitude (31) [1] [1]

d. Calculate the exposure time needed for a Keck II image of the exoplanet to have an SNR of 3. Assume
that the telescope has perfect AO, is observed at the longest wavelength for which the planet can still
be resolved from the star, all the received flux from the planet consists of photons of that longest
wavelength , = 0.1 and b = 109 photons s-1 (so b >> f). Comment on your answer.

[1]
Photon flux:
[1]

(SB-Law fplanet gives f = 25.4 photons m-2 s-1, and day side reflection increases f by a factor of 2)

Since b >> f we can simplify the denominator of the SNR formula by ignoring the first term, so
[1] [3]
This is really long (> 50 years!) so it is unlikely Keck will ever be able to directly image it [1] [1]
(alternative f gives 22.7 ks, and hence the conclusion is that it is feasible)

e. How long an exposure would JWST need in order to get the same SNR as Keck II, again if observed at
the longest wavelength for which the planet can still be resolved from the star by the telescope?
(Make similar assumptions about the received flux and use the same value of ).

(using similar reasoning to the previous part of the question)


[1]
[1]
-2 -1
(alternative method gives f = 16.5 photons m s )

Since b << f we can simplify the denominator of the SNR formula by ignoring the second term, so
[1] [3]
(alternative method gives t = 0.16 s, and day side reflection decreases t by a factor of 2)

[This is much more reasonable the optimistic assumptions we have made throughout this question
mean this is just a lower limit and so the actual exposure time will be longer, perhaps several
minutes to get a much higher SNR and to compensate for the fact for that the sensitivity is lower at
970 nm than we have quoted since it varies with wavelength. The main thing that would prevent
direct observation of the exoplanet by JWST would be if the contrast ratio was much higher than
calculated here as that would mean the star's light would need to be blocked out by the Near
Infrared Camera's coronagraph unfortunately this can only be used at wavelengths too long to
resolve the system.]
END OF PAPER
Astronomy & Astrophysics A2 Challenge
September - December 2016
Solutions and marking guidelines

The total mark for each question is in bold on the right hand side of the table. The breakdown of
the mark is below it.
There is an explanation for each correct answer for the multiple-choice questions. However, the
students are only required to write the letter corresponding to the right answer.
In Section C, students should attempt either Qu 13 or Qu 14. If both are attempted, consider the
question with the higher mark.
Answers to one or two significant figures are generally acceptable. The solution may give more
in order to make the calculation clear.
There are multiple ways to solve some of the questions, please accept all good solutions that
arrive at the correct answer.

Question Answer Mark


Section A 10
1. D 1
The Coriolis force is an inertial force that acts on objects that are in motion
relative to a rotating reference frame. As a result, hurricanes rotate anti-
clockwise in the northern hemisphere and clockwise in the southern
hemisphere. (this is also explained through cons. of angular mom.)
2. B 1
There are two tides every day: one caused by the gravitational attraction of
the Moon on the side facing it, and the other due to the centrifugal force
experienced by the side furthest from the centre of mass.
3. D 1
The number of photons collected is proportional to the area of the aperture,
which is proportional to the diameter2. Therefore if the diameter is 4 times
bigger it will receive 42 = 16 times more photons.
4. A 1
Aquila is not a zodiacal constellation. According to the astronomical
definition, there are 13 zodiacal constellations, with Ophiucus being the least
known of them.
5. D 1
The orbit of the Moon is inclined 5 to the ecliptic. During winter, the Sun has
its lowest declination. Thus the Full Moon, which is on the opposite side of
the sky, will have its highest declination and will be visible highest in the sky.
6. B 1
Each fold doubles it, so new thickness after n folds is 10 m 2n. Setting this
equal to 1 AU gives n = 53.7 so closest is 50.

1
7. C 1
The altitude of Polaris (Northern Star) above the horizon is the latitude of the
observer. Thus, at a latitude of 52 N, the altitude of Polaris is 52.
8. A 1
The rising and setting sun alignments will happen an equal number of days
before and after the winter and summer solstices respectively (roughly 21st
June and 21st December). Since 11th July is 21 days after the summer
solstice, you were looking for a date about 21 days before (in order to get a
setting Sun).
9. C 1
The first clue is that you are not at the North Pole, which would be a trivial
solution. By looking at the answers, the second clue is that it has to be close
to one of the Poles (South Pole more specifically, as the first direction of travel
is South). You start 100 Miles North of a line of latitude whose entire length is
100 Miles. You travel South 100 Miles, then East 100 Miles until you encircle
the South Pole completely, then travel back North 100 Miles to finish where
you started. The radius of a circle with the circumference of 100 Miles is
16 Miles. Thus, you have to start 116 Miles from the South Pole. This is
only an approximate answer, as it assumes the Earth to be flat. However, the
Earth does not curve much on the small distances we considered in the
question, a full treatment on a sphere would give an answer just 7 cm away.
10. B 1
.
The angular size of a pixel is 0.0011 1.9 10

The distance the image was taken at is: !" 2.1 km
.
#$

Section B 10
11. a. 2
i. Answer: 463 m s

Sun will appear stationary if the Eurofighter moves at the Earth's rotational
speed, so:

2012 20 6.37 106


()*+,-./ 463 m s
3 24 60 60 1

ii. Answer: 285 m s

Oxford is less far from the Earth's rotational axis, so the speed needed will be
less by a factor of cos (where is the latitude of Oxford)

20 12 cos =
(.89./: 285 m s-1
1
3

b. 3

2
At the Equator a day is 12 hours (due to the value of ()*+,-./ ), and during a
day the Sun travels 180, so since it is 0.5 across it covers that angular
distance in:

. 1
12 = 0.033 hours = 2 minutes
@

In the Eurofighter the effective Earth rotation speed becomes:


500 463 = 37 G H = 0.08 ()*+,-./ 1

So time to rise the sun by 0.5 becomes:


= 25 minutes
minutes 1
. @

12. a. Answer: 12 900 km 3

Need ratio of diameter (or radius) and distance to be the same, so:
I M
=
KL : 1
d = 12 900 km
1
This is a much higher altitude than the ISS.
1

[Note 1: if the student forgets to convert both into radii or into diameters (i.e.
gets d = 25 800 km) then they lose the first mark, but can get 2 ecf marks]
[Note 2: students can use the comparison with angular diameter of 0.5 from
the previous question for the first mark, but must recognise that they need to
convert the angle into radians to get the second mark (if done correctly they'll
get d = 13 800 km)]

b. Answer: 7.41 hours 2


Using Kepler's Third Law:

N
3 = QR
OP

N N
1
3=T V12 + XYR = T [[ . Z N\
V6.37 106 + 12.9 106 YR
OPU 6.6Z

1
T = 2.67 104 s (= 7.41 hours)

[Note 1: first mark is for recognition that the radius of the orbit is equal to the
radius of the Earth plus the altitude, expressed either algebraically or through
the substitution]
[Note 2: allow ecf from part a. so long as value for final period is sensible]

3
Section C Either Qu 13 or Q 14 10
13. a. 4
At Roche limit Fgrav = Ftidal, so:

Oa+ OP+/
/N
= h
:fg

Correct rearrangement and cancelling:

[
P h 1
XI] = ^ i2 j
a

Mass of planet and satellite in terms of density:

1
But k = 0`P 1 and G = 0`a ^
R R
R R

Substitution of densities into equation:

`P 1R R 1
XI] ^ _2 b
`a ^ R

Cancelling of r to leave required expression:

`P /R
XI] = 1 c2 d
`a
1

b. 2
kl,-+/m 5.68 10 6
`l,-+/m = = = 619 kg m-3
4 R 4
01 0 V6.027 10Z YR 1
3 l,-+/m 3

`l,-+/m /R 619 /R
XI] = 1l,-+/m c2 d = 60 270 c2 d = 66 300 km
`pq) 930
V= 1.10 1l,-+/m Y 1

c. 1
619 /R
XI],a,8 = 2.44 60 270 c d = 128 400 km V= 2.13 1l,-+/m Y
1
930
d. 1
The limits (roughly) agree with the observed extent of the ring system 1
[Allow students to say the inner edge agrees well with the simple model, but
the outer edge is a poorer fit]

e. 2
Finding the density of a moon that reached the fluid Roche limit at 2 1l,-+/m :

st 6
`a = V Yh
=V Yh
= 1124 kg m-3
/ . / . 1

4
Assuming Veritas was spherical then the radius is:

/R /R
Puvwx R [z
^ _\ b _\ b 1.85 10 m V= 185 kmY 1
sy
h h

[Given that this is similar in size to several other moons of Saturn, the idea
that the rings came from the tidal destruction of a moon is not completely
outlandish]

14 a. 1
The period is the time interval between two consecutive peaks of the blue
curve. From the radial velocity curve, the period is 11 days.
1
[Full marks for the period within 2 days]

b. 1
Using Kepler's Third Law:

~N N

,h
= OP

The semi-major axis of the planets orbit is:

/R
{k 3
Q= _ b = 7.16 10 m = 0.048 AU 1
40

[Note: allow ecf from part a. so long as value for the semi-major axis is
sensible]

c. 1
The orbital velocity for a circular orbit is:

20Q
(qp/q =
3

(qp/q = 47.30 km s 1
[Note: allow ecf from part a. so long as value is sensible]

d. 3
The total linear momentum in the centre of mass frame is: P = 0

-,/ + ,m)- = 0

k(-,/ G(,m)- = 0

(-,/
G=k 1
(,m)-

Using (-,/ = 5 km hour = 0.0014 km s and (,m)- = 47.30 km s :

0.0014
G = 0.12 2 10R = 7.1 10 kg = 1.19 k2,/-
47.3 1

This is a lower estimate for the mass of the planet because the inclination of

5
the planets orbit is not known. Astronomers are only able to measure the
radial velocity of the star, not the tangential one. In the calculations above we
assumed that the plane of the orbit is in the line of sight, thus the radial 1
velocities are the maximum, total velocities. Therefore, the mass we
determined is a minimum mass for the planet; it is in fact G sin , where is
the inclination of the orbit.
e. 3
We are given that the orbit is an ellipse, hence using the diagram on page 2,
we can determine the minimum and maximum distance from the planet to
the star.
The minimum distance (also known as periapsis) is:

^apm = QV1 Y

^apm = 0.048V1 0.35Y = 0.031 AU

The maximum distance (also known as apoapsis) is:

^a,8 = QV1 + Y

^apm = 0.048V1 + 0.35Y = 0.065 AU

Thus, the distance from the planet to Proxima Centauri ranges between 1
0.031AU and 0.065 AU.

Hence the maximum equilibrium temperature of the planet is:

1l-,/ /
3,m)-,a,8 = 3l-,/ c d
2^apm

3,m)-,a,8 = 307 K

The minimum equilibrium temperature of the planet is:

1l-,/ /
3,m)-,apm = 3l-,/ c d
2^a,8

3,m)-,apm = 212 K

The temperature of the planet ranges between 212 K and 307 K, or -61C and 1
34C.

The habitable zone is the band around a star where a planet can have water
on its surface in liquid form, at normal pressure. Hence, the equilibrium
temperature of the planet must be between 0C and 100C. The temperature 1
on Proxima Centauri B reaches values above 0C, thus it is in the habitable
zone of its host star.

f. 1
The distance to Proxima Centauri, the nearest star to the Sun, is 4.22 light
years. Assuming that the lifetime of a human being is 80 years, the robotic
space probe would need to travel at a speed of 15 800 km s to reach the
star in a lifetime. Such high velocities have not been achieved yet, currently 1
the fastest man-made object is the Juno Mission, travelling at 40 km s .
Thus, the prospects of reaching Proxima Centauri B in our lifetime are low,
unless new technology is developed.

6
Astronomy & Astrophysics A2 Challenge

September December 2016

Instructions

Time Allowed: One hour

In Section C, you can choose to answer either Q13 on the Saturns rings, or Q14 on
the discovery of the nearest exoplanet.

Marks allocated for each question are shown in brackets on the right.

You will need to use a ruler.

You may use any calculator.

You may use any standard formula sheet.

This is the first paper of the British Astronomy and Astrophysics Olympiad in the 2016/2017
academic year. To progress to the next stage of the BAAO, you must take BPhO Round 1 in
November, which is a general physics problem paper.

To be awarded the highest grade (Distinction) in this paper, it should be sat under test conditions
and marked papers achieving 60% or above should be sent in to the BPhO Office at Oxford by
Wednesday 9th Nov 2016.

To solve some of the questions, you will need to write equations, draw diagrams and, in general,
show your working. You are also encouraged to look at the clear sky and identify the brightest
stars, a few days before sitting the paper.

This paper has more than an hours worth of questions. You are encouraged to have a go at as
many as you can and to follow up on those that you do not complete in the time allocated.
British Physics Olympiad Sponsors

Worshipful Company of Scientific


Instrument Makers

1
Useful constants

Speed of light c 3.00 10 ms


Gravitational constant G 6.67 10 N m kg
Solar mass M 1.99 10 kg
Solar radius R 6.95 10 m
Astronomical Unit AU 1.496 10 m
Light year ly 9.46 10 m
Earths orbit semi-major axis 1 AU
Earths orbital period 1 year 365.25 days
Earths rotation period 1 day 24 hours
Earths mass MEarth 5.97 10 kg
Earths radius REarth 6.37 10 m
Earths axial tilt 23.4

You might find the diagram of an elliptical orbit below useful in solving some of the
questions:
B

P a A
F O

C
Elements of an elliptic orbit:
= OA (=PO) - semi-major axis
= OB (=CO) - semi-minor axis
"#
= 1 $# - eccentricity
F - focus
P periapsis (point nearest to F)
A apoapsis (point furthest from F)

Keplers Third Law:

For an elliptical orbit, the square of the period (&) of orbit of an object about the focus
is proportional to the cube of the semi-major axis ( ) (the average of the minimum and
maximum distances from the Sun). The constant of proportionality is 4( )*, where M
is the mass of the central object.

The questions were proposed by:

Dr Alex Calverley (Bedford School)


Sandor Kruk (University of Oxford)

2
Section A: Multiple Choice

Write the correct answer to each question. Each question is worth 1 mark. There is only
one correct answer to each question. Total: 10 marks.

1. Why do hurricanes rotate anti-clockwise in the northern hemisphere and clockwise


in the southern hemisphere?
A. Due to the Earth rotating from East to West
B. Due to the different ratios of land to water area between the two
hemispheres
C. Due to the Moon's orbit being inclined by 5 above the ecliptic giving it
more influence on the northern hemisphere
D. Due to the Coriolis Effect causing paths of particles to curve as they travel
over the Earth's surface

2. Why are there two tides every day?


A. The Moon causes the one at night and the Sun causes the one during the
day
B. The Earth and Moon orbit a common centre of mass
C. The Moon is tidally locked to the Earth
D. Water waves can only travel around the Earth in 12 hours

3. A typical cheap handheld telescope has a diameter of 10 cm, whilst ones for keen
amateurs can have diameters of 40 cm. How much greater light gathering power does
the larger telescope have?
A. 2
B. 4
C. 8
D. 16

4. Which of the following is not a zodiacal constellation, according to the astronomical


definition?
A. Aquila B. Aquarius C. Ophiucus D. Pisces

3
5. When observing from the UK, during which season is the Full Moon visible highest
in the sky?
A. Spring
B. Summer
C. Autumn
D. Winter

6. Given an overwhelmingly large piece of paper, with a thickness of 10 m,


approximately how many times do you need to fold it in half (theoretically!) for the
thickness of the final stack to reach from Earth to the Sun (1 AU)?
A. 40
B. 50
C. 60
D. 70

7. At a latitude of 52 N what is the altitude of Polaris above the horizon?


A. 38
B. 48
C. 52
D. 90

8. "Manhattanhenge" is the name given to when, just before sunset or just after sunrise
4 times a year (twice for setting, and twice for rising), the Sun aligns with the east-west
streets of the New York grid system. One of the setting dates this year was on 11th July.
Which of these is another date you are likely to see the "Manhattanhenge" sunset?
A. 30th May
B. 30th June
C. 11th December
D. 11th January

9. You travel 100 miles South, 100 miles East and 100 miles North and arrive back
where you started. Where are you? You are NOT at the North Pole.

A. South Pole
B. 100 Miles from the North Pole
C. 116 Miles from the South Pole
D. 200 Miles from the South Pole

10. On 5th September 2016, the Rosetta mission has finally found the Philae lander on

an image from the high-resolution camera (with 2048 2048 pixels and field of view
Comet 67P/ChuryumovGerasimenko. Considering that Philae (111 m) appeared in

2.2 2.2) as 25 25 pixels, from what distance did Rosetta manage to image Philae?

A. 1.1 km
B. 2.1 km
C. 12.2 km
D. 26.8 km

4
E. Section B: Short Answer

Each short question is worth 5 marks. Total: 10 marks.

Question 11 Forces of Nature

In the BBC programme Forces of Nature, Brian Cox uses a Eurofighter Typhoon to try
and overtake the spin of the Earth such that the setting Sun appears to rise instead.

a. By considering the circumference of the Earth at each point, and the


length of a day, what speed in the air would the Eurofighter need to
achieve for the Sun to appear stationary if it took off from:
i. The equator
ii. Oxford (which has a latitude of about 52)
[You can ignore the altitude of the fighter jet]
[2 marks]

b. A Eurofighter sets off from the equator just as the top edge of the Sun has
gone below the horizon, and rapidly accelerates due west up to a speed
of 500 m s-1. Given that the Sun has an angular diameter of 0.5 as
viewed from Earth, what is the minimum amount of time the fighter jet
needs to fly for in order to see the whole of the Sun above the horizon?
[3 marks]

5
Question 12 Star Wars Rogue One

The new Star Wars film Rogue One concentrates on the creation of the first Death Star,
which in one scene causes a total eclipse on the planet Scarif, where it is being built.

a. Assume the Death Star is being built in orbit around the Earth instead, but
still causes a very brief total solar eclipse when it passes in front of the
Sun. The Death Star has a diameter of 120 km, and so by comparing it to
the size and distance to the Sun, calculate the altitude it is being built at.
How does that compare to the altitude of the International Space Station
(400 km)?
[3 marks]

b. What will be its orbital period, assuming it moves in a circular orbit?

[2 marks]

6
Section C: Long Answer
Each long question is worth 10 marks. Answer either Qu 13 or Qu 14. Total: 10 marks.

Question 13 Saturns Rings

One possible theory for why the gas giants have ring systems is that a small moon got
too close to the parent planet. When the gravitational tidal forces (due to the difference
between the strength of the planet's pull on the near and far sides of the moon) became
greater than the gravitational forces holding the moon together, it was ripped apart. This
minimum distance is called the "Roche limit", named after the French astronomer
Edouard Roche who first calculated it. It is defined as when the gravitational force
generated by the moon at its surface is equal to the tidal forces it experiences at that
distance.

Cassini division

Consider a spherical planet with mass * and radius ,, and a perfectly rigid spherical
moon with mass - and radius ., orbiting the planet in a circular orbit of radius /. For a
small particle of mass 0 on the surface of the moon, the gravitational and tidal forces it
experiences will be

)-0
123$4 =
2)*0.
.
1567$8
/

a. By making these two expressions equal, derive an expression for the


Roche limit, /9: , purely in terms of , and the uniform densities of the
planet and the moon (;< and ;= respectively)
[4 marks]

7
b. Use your formula to calculate the Roche limit of Saturn for a moon made
of water ice (;= = 930 kg m-3), given that *>$5?3@ = 5.68 1026 kg and
,>$5?3@ = 60 270 km
[2 marks]

c. In practice, as a moon approaches the Roche limit it will start to deform


and become more of an ellipsoid than a sphere, causing the tidal forces
to increase, and so the Roche limit from our simple model is really a
minimum radius. The opposite extreme would be to assume that both the
planet and moon are made of a fluid, and so can deform without
resistance (this works well when looking at things like stars in close
binary systems). In that situation it can be shown that the equivalent
formula for the Roche limit becomes

;C
/9: 2.44, B D
;=

Work out this new maximum value for the Roche limit for water ice
around Saturn.
[1 mark]

d. The inner edge of Saturn's rings (D ring) occurs at 1.11 ,>$5?3@ , and the
outer edge of the A ring (the last main visible ring) is at 2.27 ,>$5?3@ . Do
the rings of Saturn fall (roughly) between the two Roche limits calculated
for the extreme cases of a perfectly rigid and a fluid moon made of water
ice?
[1 mark]

e. Edouard Roche was one of the first scientists to suggest the destruction of
a moon (which he named Veritas) as a source of Saturn's rings. Assume
that it used to orbit in what is now the Cassini Division (an apparent gap
in the rings at around 2 ,>$5?3@ ), and that the fluid Roche limit was most
relevant in this case. Given that the mass of the rings is 3.0 1019 kg,
estimate the radius of Veritas.
[2 marks]

8
Question 14 Nearest exoplanet discovered

On 24th August 2016, astronomers discovered a planet orbiting the closest star to the
Sun, Proxima Centauri, situated 4.22 light years away, which fulfils a long-standing
dream of science-fiction writers: a world that is close enough for humans to send their
first interstellar spacecraft.

Astronomers have noted how the motion of Proxima Centauri changed in the first
months of 2016, with the star moving towards and away from the Earth, as seen in the
figure below. Sometimes Proxima Centauri is approaching Earth at 5 km hour-1
normal human walking pace - and at times receding at the same speed. This regular
pattern of changing radial velocities caused by an unseen planet, which they named
Proxima Centauri B, repeats and results in tiny Doppler shifts in the stars light, making
the light appear slightly redder, then bluer.

a. From the radial velocity curve above, determine the period of the
planet around Proxima Centauri.
[1 mark]

b. Proxima Centauri is a red dwarf star, unlike our Sun, with a mass of
only 0.12 M . What is the semi-major axis of the planets orbit in AU?
[1 mark]

c. Assuming that the orbit of Proxima Centauri B is circular, what is the


planets orbital velocity?
[1 mark]

d. By considering that the total linear momentum of the star-planet


system in the centre of mass frame is zero, estimate the minimum
mass of the planet in terms of Earth masses. Why is this a minimum
for the mass of the planet?
[3 marks]

9
e. Using a simple approximation, the equilibrium temperature of a
planet can be calculated as
,>5$3
&<8$@E5 = &>5$3 F
2/

where / is the distance between the star and the planet. Given that
the astronomers discovered that the orbit of the planet is in fact an
ellipse with an eccentricity of 0.35, and that the star has a surface
temperature of 3000 K and a radius of 0.14 R , what are the
minimum and maximum equilibrium temperatures of Proxima
Centauri B?

Comment on whether or not the planet is in the habitable zone of


Proxima Centauri.

[The habitable zone is the band around a star where a planet can
have water on its surface in liquid form, at normal pressure.]
[3 marks]

f. Comment on the prospects of studying the planet directly, during your


lifetime, using robotic space probes.
[1 mark]

END OF PAPER

10
British Astronomy and Astrophysics Olympiad 2015-2016

Astronomy & Astrophysics Competition Paper

Monday 18th January 2016


Instructions
Time: 3 hours (approximately 35 minutes per question).
Questions: All five questions should be attempted.
Marks: The questions carry similar marks.
Solutions: Answers and calculations are to be written on loose paper or in examination
booklets. Students should ensure their name and school is clearly written on all answer sheets
and pages are numbered. A standard formula booklet may be supplied.
Instructions: To accommodate students sitting the paper at different times, please do not
discuss any aspect of the paper on the internet until 8 am Saturday 23rd January.
Clarity: Solutions must be written legibly, in black pen (the papers are photocopied), and
working down the page. Scribble will not be marked and overall clarity is an important aspect
of this exam paper.

Eligibility: The International Olympiad will be held during December 2016; all A Level students
are eligible to participate, even if they will be attending university in December.

--- ------------------------------ ----------------------------------- ---------------------------- ---

Training Dates and the International Astronomy and Astrophysics Olympiad


Following this round the best students eligible to represent the UK at the International
Olympiad in Astronomy and Astrophysics (IOAA) will be invited to attend the Training Camp to
be held in the Physics Department at the University of Oxford, (Monday 4th April Thursday 7th
April 2016). Astronomy material will be covered; problem solving skills and observational skills
(telescope and naked eye observations) will be developed. At the Training Camp a practical
exam and a short theory paper will be sat. Five will be selected for further training. From May
there will be mentoring by email to cover some topics and problems, followed by a training
camp at the beginning of July and a weekend training camp in autumn.

The IOAA this year will be held in Bhubaneswar, India, from 9th to 19th December 2016.
Important constants

Speed of light in free space c 3.00 10 m s


Earths rotation period 1 day 24 hours
Earths orbital period 1 year 365.25 days
Parsec pc 3.09 10 m
Astronomical Unit AU 1.49 10 m
Radius of the Earth 6.37 10 m
Radius of the Earths orbit 1 AU
Radius of the Sun 6.96 10 m
Mass of the Sun 1.99 10!" kg
Mass of the Earth % 5.97 10&' kg
Luminosity of the Sun ( 3.85 10& W
Gravitational constant + 6.67 10 m! kg s &

You might find the diagram of an elliptical orbit below useful in solving some of the questions:
B

P A
F O a

C
Elements of an elliptic orbit: a = OA (=PO) semi-major axis
b = OB (=CO) semi-minor axis
e = ,1 0 /
./
eccentricity
F focus
P periapsis (point nearest to F)
A apoapsis (point furthest from F)
Keplers Third Law: For an elliptical orbit, the square of the period, T, of orbit of an object about the focus is

from the Sun). The constant of proportionality is 41 & /+ , where is the mass of the central object.
proportional to the cube of the semi-major axis, a (the average of the minimum and maximum distances

Magnitudes: The apparent magnitudes of two objects, 3 and 3" , are related to their apparent
brightnesses, 4 and 4" , via the formula:
4
= 10 ".'(78 79 )
4"

2
Supported by:

BPhO sponsors

Random House Publishers

Cambridge University Trinity College

Cavendish Laboratory

3
Qu 1. Asteroid Belt
In science fiction films the asteroid belt is typically portrayed as a region of the Solar System where the
spacecraft needs to dodge and weave its way through many large asteroids that are rather close together.
However, if this image were true then very few probes would be able to pass through the belt into the outer
Solar System.

Figure 1 Artist conceptual illustration of the asteroid belt (left). Schematic of the Solar System with the asteroid
belt between Mars and Jupiter (right).

This question will look at the real distances between asteroids.

a. Given that the total mass of the asteroid belt is approximately ;<=> = 1.8 10 ? calculate the
radius of the object that could be formed, assuming it has a density typical of rock 6@ 5 3.0 g cm ! ).
Compare this to the radius of the largest member of the asteroid belt, Ceres. ( B<C<D = 473 km)
b. The main part of the asteroid belt extends from 2.1 AU to 3.3 AU, and has an average angular width
of 16.0, as viewed from the Sun. Calculate the average thickness of the belt, and hence its total
volume, E;<=> .
c. Assuming this volume is uniformly filled by spherical rocky asteroids of average radius 0F , derive a
relationship between the average distance between asteroids, G0F , and their radius 0F ,
remembering to keep the total mass equal to ;<=> .
d. If 0F = 2.0 km, calculate G0F . How does this compare to the Earth-Moon distance?
(G I 5 384,000 km)

An object with an apparent magnitude 3" 5 0 has an apparent brightness 4" 5 2.52 10 W m &.

e. Using the luminosity of the Sun, calculate the total power incident on an asteroid in the middle of
the asteroid belt.
f. Assuming only 30% of that is reflected by its rocky surface, calculate the apparent magnitude of the
asteroid when viewed from its nearest neighbour. Given that objects with 3 J 6 are too faint for
the naked eye, would it be visible to an astronaut stood on the asteroid surface?

4
Qu 2. Supermoons

Figure 2 Supermoons at Perigee and Apogee. Image credit: John Gaughan/Pete Lardizabal/ WJLA.

A supermoon is a new or full moon that occurs with the Moon at or near its closest approach to Earth in a
given orbit (perigee). The media commonly associates supermoons with extreme brightness and size,
sometimes implying that the Moon itself will become larger and have an impact on human behaviour, but
just how different is a supermoon compared to the normal Moon we see each month?

Lunar Data:
Synodic Period = 29.530589 days (time between same phases e.g. full moon to full moon)
Anomalistic Period = 27.554550 days (time between perigees i.e. perigee to perigee)
Semi-major axis = 3.844 10K km
Orbit eccentricity = 0.0549
Radius of the Moon = 1738.1 km
Mass of the Moon = 7.342 10&& kg

In this question, we will only consider a full moon that is at perigee to be a supermoon.

a. Calculate how many days separate two supermoons.


b. Show that the difference in distance between the apogee and perigee is 4.22 10' km.
(The data given in this question allows the mean orbital parameters to be calculated. Note that
perturbations in the lunar orbit mean that the perigee and apogee continually change over the
course of the year.)
c. Determine the difference in the angular diameter of a supermoon and a full moon observed at
apogee. Thus, determine the percentage difference in the brightness of a supermoon and a full
moon observed at apogee. (Ignore the effects of the Moons orbital tilt with respect to the
Earth.)
d. What change in magnitude does this brightness difference correspond to?
e. Suggest why it can be difficult to detect any differences in the brightness of supermoon
compared to a normal full moon when observing with the naked eye?
f. Calculate the gravitational field of the supermoon at the Earth. What fractional mass increase
would a Moon at apogee need in order to create the same gravitational field?

5
Qu 3. Interstellar

In the science fiction movie Interstellar, the crops on Earth are failing, making farming difficult, and the
existence of humanity is threatened. To save the human race, a crew of astronauts travelled through a
wormhole in search of a new home and they sent encouraging data from planets near Gargantua, a
supermassive black hole:
Millers planet is the first planet in the system orbiting Gargantua. It is a water world with a similar
composition to the Earth, covered in an endless shallow ocean. The planet's gravity is 130% of the Earth's,
forcing human astronauts to move slowly and with some difficulty while on its surface. Being well within the
tremendous gravitational field of Gargantua, time on the surface of Miller's planet passes very slowly
relative to the rest of the universe: a single hour on Miller would equate to seven years back on Earth.
Because of the planet's proximity to Gargantua, the immense gravitational pull from the black hole causes
the planet to be afflicted by massive tidal waves as tall as 1 km. There is no sign of dry land on Miller, which
may not exist due to the enormous erosive power of the planet's waves.

Figure 3 CGI model of a supermassive black hole and Millers Planet. Credit: Interstellar.

The gravitational time dilation is given by:

L" = LM N1 -
2+
O&

where L" is the time for a slow-ticking observer within the gravitational field, LP , is the time for a fast-ticking
observer at an arbitrarily large distance from the massive object, is the mass of the massive body, and is
the distance of the observer from the centre of the body.

You are asked to estimate the following:


a. The characteristics of the planet (mass and radius).
b. The mass of the supermassive black hole.
c. The orbital parameters of the planet (orbital radius and period).
d. The planet orbits the black hole, but the Hollywood director seems not to have checked his
numbers carefully. In what way is this apparent from the values given and the results you
have calculated? (the radius of the black hole is the event horizon, the value of when
LP .)

6
Qu 4. Dyson structures
Since its first light in 2009, the Kepler Telescope has been scanning the universe in search of habitable worlds
beyond our Solar System. Kepler is designed to observe stars and look for tiny dips in their brightness. These
dips, especially if they repeat, can be a sign the star has planets orbiting it. By measuring the timing and the
size of the dips, scientists can learn a lot about the transiting planet.

During its routine observations of the star KIC 8462852, similar to our Sun (same radius and mass), the
telescope observed something very unusual. A group of citizen scientists noticed that this star appeared to
have two small dips in 2009, followed by a large dip lasting almost a week in 2011, and finally a series
of multiple dips significantly dimming the stars light in 2013. The pattern of the dips indicates that a large,
irregular-shaped object orbits the star. Some people have speculated that the star might be orbited by a
giant alien megastructure, called a Dyson structure. It is a structure that harnesses a stars energy to be
used by a civilisation, like solar power, but on a massive scale. It would be composed of thousands of
spacecraft that would be theoretically large enough to block out a significant portion of a star's light.

Figure 4 The light curves of KIC 8462852 showing two transits. The time is in days since a reference point. Credit:
Boyajian et al, 2015, Planet Hunters X. KIC 8462852 - Where's the Flux?

a. Explain why the scientists believe that the object is unlikely to be a star or a planet.
b. Judging from the light curves in the plots above, what would be the area of the Dyson
structure?
c. Based on the largest dips, what is the average distance of the Dyson structure from the star?
Assume its mass is much smaller than the mass of the central star.

The problem of creating a Dyson structure is that it cannot be free floating in space. One possible solution is

inwards, and the light pressure pushing them outwards. The luminosity (power output) of the star is ( .
creating a cloud of solar sails. These objects would be in perfect balance between the gravity pulling them

d. Assuming that the sails are made of a reflective material with reflectivity R, what is the
pressure on the sail due to photon bombardment? (momentum of a photon is E/c)
e. What is the force exerted by the photons on the Dyson structure? Assume R = 1.
f. Assuming that the net acceleration of the solar sails is zero, what would be the mass of the
structure?
g. Explain why the scenario of building such a structure is unrealistic.

7
Qu 5. Gravitational lensing

The deflection of light by a gravitational field was first predicted by Albert Einstein a century ago, suggesting
that massive objects can bend light like a classical lens. This prediction was confirmed by Sir Arthur Edington
in 1919, while observing a solar eclipse.

Consider a spherically symmetric object with mass M. This object will act like a lens, with an impact
parameter b measured from the centre of the object. The angle of deflection due to the massive lens, given
by General Relativity, is calculated as:

R=
4+
4O &

In a simplified model, the impact parameter may be seen as the shortest separation between the centre of
the lens and the path of a particular light ray. The diagram below shows the geometric model of a
gravitational lens (Figure 5). Light rays emitted from the source S being deflected by the lens are observed as
images S1 and S2. The angles are very small.

S1

S2

Figure 5 Schematic diagram of a gravitational lens.

a. Explain how Arthur Eddington might have used the gravitational lens effect to confirm the
predictions of General Relativity.
b. Show that the source angle ST is related to SU , , VT , VX via the expression:

VXT 4+
ST Y SU -
VX VT O & SU

c. For the special case in which the source is perfectly aligned with the lens such that SZ 5 0,
a ring-like image (called an Einstein ring) will occur. Find the angular radius, called the
Einstein radius S% , of the ring.

8
d. Show that the time delay for a photon in the presence of an Einstein Ring is given by:

L =
1 VT VX &
S
2 VXT O %

1 R&
]You may use the approximation: 5 1 g h
cos 2

Extending the example of gravitational lensing into 3 dimensions, instead of two images of the source,
sometimes multiple images of the source can be seen, arranged in a cross (called an Einstein cross). Last
year, astronomers discovered a galaxy that is gravitationally lensed by a giant elliptical galaxy situated in a
galaxy cluster in the foreground. Surprisingly, they discovered a supernova explosion (called the Refsdal
Supernova) in the image of the lensed galaxy, arranged in an Einstein cross.

Figure 6 Multiply-lensed Refsdal supernova by a massive galaxy in the galaxy cluster MACS
J1149.6+2223. Credit: Hubble/NASA/ESA/STSci/UCLA.

e. By knowing that the distance to the galaxy is 4.4 Gpc, and to the cluster is 2.0 Gpc, find the
time delay caused by the lens for a photon from the supernova explosion. You can take the
mass of the elliptical galaxy to be = 1012 . A Gpc is 109 parsecs.
f. Perhaps even more surprisingly, the astronomers realised that they were seeing the four
images of the supernova at different time instances. In some of the pictures they took,
images of the supernova were missing. Explain how this is possible.

End of Questions

9
Asteroid Belt (solutions)

a. Given that the total mass of the asteroid belt is approximately Mbelt = 1.8 10-9 M, calculate the
radius of the object that could be formed, assuming it has a density typical of rock ( = 3.0 g cm-3).
Compare this to the radius of the largest member of the asteroid belt, Ceres. (RCeres = 473 km)

Mbelt = 1.8 10-9 M = 3.6 1021 kg


density = 3.0 g cm-3 = 3.0 103 kg m-3
.
= = = .
[1]

= 660 km [1]

= 1.4 Ceres [1]

[This means Ceres contains a sizeable fraction of the material in the whole belt, although it has a
lower density than we have assumed, so less mass than implied here]

b. The main part of the asteroid belt extends from 2.1 AU to 3.3 AU, and has an average angular width
of 16.0, as viewed from the Sun. Calculate the average thickness of the belt, and hence its total
volume, Vbelt.

h
8
2.1 AU 3.3 AU
. .
Average height above the orbital plane, h = tan 8 = 0.38 AU [0.5]

So total thickness = 2h = 2 0.38

= 0.76 AU [0.5]

Volume of belt = area of disk total thickness

= 3.3 2.1 0.76 [1]

= 15.45 AU

= 5.1 10 m [1]

= 5.1 10 km

[This is a huge volume, which explains why each asteroid gets so much space to itself]

c. Assuming this volume is uniformly filled by spherical rocky asteroids of average radius Rav, derive a
relationship between the average distance between asteroids, dav, and their radius Rav, remembering
to keep the total mass equal to Mbelt.

belt
Assuming there are N asteroids, each filling a cube with side length dav then = [1]
av

belt
Conserving the volume of rock in the belt means av = [1]
so we can cancel N to give
belt belt
av =
av

av belt
= [1]
av belt

(a spherical approximation for the volume of belt allocated to each asteroid can also gain full credit
so long as it is clear that dav in that case is equal to double the radius of the spherical volume used)
belt av belt
i.e. = leading to =
av av belt

d. If Rav = 2.0 km, calculate dav. How does this compare to the Earth-Moon distance? (dEM = 384,000
km)

av .
av = =
belt .
belt . .

= 1.1 10 m ( = 1.1 10 km) [1]

= 2.9 EM [1]

(using a spherical approximation in the previous question yields dav = 1.4 109 m = 3.6 dEM)

[An average separation of about 1 million km is close to the real value for our asteroid belt and
emphasises the vast space between asteroids, which is why many probes can travel through
unharmed, although the real value of Rav is somewhat smaller this is because the real asteroid belt
contains far more small asteroids than big ones]

e. Using the luminosity of the Sun, calculate the total power incident on an asteroid in the middle of the
asteroid belt.

. .
Distance to middle of the belt, mid = = 2.7 AU
Apparent brightness of the Sun in the middle of the belt:
.
= = . .
[1]
mid

= 190 W m-2 [1]

Incident power, Pi = apparent brightness cross-sectional area of an asteroid

i = av = 190 2.0 10 [1]

= 2.4 10 W [1]
f. Assuming only 30% of that is reflected by its rocky surface, calculate the apparent magnitude of the
asteroid when viewed from its nearest neighbour. Given that objects with m > 6 are too faint for the
naked eye, would it be visible to an astronaut stood on the asteroid surface?

Apparent brightness of one asteroid as viewed from another with only 30% reflectivity:

. . .
= = .
[1]
= 4.5 10 W m-2 [1]

.
= .
log = 2.5 log .
[1]

= 6.9 [1]

This means that the astronaut could not see the nearest asteroid with their naked eye [1]

(Using a spherical approximation gives b = 2.9 10-11 W m-2 and m = 7.3, so the same conclusion)

[In practice even though asteroids come in a wide variety of sizes, and the spacing can vary quite far
from our values of dav, most asteroids only reflect about 10% of their light and so despite all the
simplifying approximations we have made we still get the same result that without binoculars you
would be unable to tell you were flying through an asteroid belt, which is rather different to the
picture painted by science fiction films!]
1

Figure 1: Image credit: John Gaughan / Pete Lardizabal / WJLA

Supermoons
The termsupermoon was coined by astrologer Richard Nolle in 1979. He
defined a supermoon as a new or full moon that occurs with the Moon at or near
(within 90% of) its closest approach to Earth in a given orbit (perigee). The
value of 90% was arbitrarily chosen, and other definitions are often used.
The media commonly associates supermoons with extreme brightness and
size, sometimes implying that the Moon itself will become larger and have an
impact on human behaviour, but just how different is a supermoon compared to
the normal Moon we see each month?

Lunar Data:
Synodic Period = 29.530589 days - time between same phases e.g. full moon
to full moon, new moon to new moon.
Anomalistic Period = 27.554550 days - time between perigees e.g. perigee
to perigee.
Semimajor axis (a)= 3.844 105 km
2

Orbit eccentricity (e)= 0.0549

Radius of the Moon (R% )= 1738.1 km

Mass of the Moon (M% )= 7.342 1022 kg


In this question, we will only consider a full moon that is at perigee to be a
supermoon.

a) Calculate how many days separate a supermoon.

b). Show that the mean difference in the distance between the apogee and
perigee is 4.22 104 km. (The data given in this question allows the mean orbital
parameters to be calculated. Note that in reality, perturbations in the lunar orbit
mean that the perigee and apogee continually change over the course of the year).

c). Determine the difference in the angular diameter of a supermoon and a


full moon observed at apogee. Thus, determine the percentage difference in the
brightness of a supermoon and a full moon observed at apogee. (Ignore the effects
of the Moons orbital tilt with respect to the Earth).

d). What change in magnitude does this brightness difference correspond to?

e). Suggest why it can be difficult to detect any differences in the brightness
of a supermoon when observing with the naked eye?

f). Calculate the gravitational force of the supermoon on the Earth. What
mass increase would a Moon at apogee need, to create the same gravitational
force?

Solutions
a). 27.554550/(29.530589 27.554550) = 13.9443 synodic periods between
each supermoon,

so 13.9443 29.530589 = 411.78 days between each supermoon.

b). From equation of ellipse we have that: Ra = a(1 + e) and Rp = a(1 e),
where Ra is the lunar apogee, Rp is the lunar perigee, a is the semi-major axis
and e is the eccentricity.
Therefore the difference in apogee and perigee is: Ra Rp = a(1+e)a(1e) =
2ae = 4.22 104 km.
3

c). Angular diameter of a full moon at apogee: R% /Ra = 4.286 103

Angular diameter of a full moon at perigee: R% /Rp = 4.784 103

So the Moon appears 4.784 103 /4.286 103 = 1.116 times or 11.6% larger
at perigee.

Since light will follow an inverse square law, the brightness will decrease
according to the distance2 . Hence, in terms of brightness, a Moon appears
1.112 = 1.246 times or 25% brighter at perigee.

d). Difference in magnitude = log10 (difference in brightness)/0.4


= log10 (1.246)/0.4 = 0.239 magnitudes.

e). Typically, a difference of 0.239 magnitudes should be detectable with the


naked eye. However, several months separate these two extremes of the bright-
est and dimmest full moons. In addition, the Moon is passing through different
phases (e.g. crescent, quarter, gibbous) as it reaches full Moon, all of which have
different brightnesses. Therefore, it is unlikely that a naked eye observer will
notice, and furthermore remember, the difference in brightness between a super-
moon and a full moon at apogee.
G M M
f). Gravitational force of Moon at perigee: Fgrav,p = %,
Rp2
G M M
and gravitational force of Moon at apogee: Fgrav,a = %
2
Ra
G M M
or write as: Fgrav,a = % +m .
(Rp +r)2

But from part b), know that r = 4.22 104 km.

If Fgrav,p = Fgrav,a , the increase in lunar mass, M% + m = 1.246M% , or


m = 0.246M% . (since gravity also follows the inverse square law, this is analagous
to the answer in part c).
British Olympiad in Astronomy and Astrophysics

Competition Paper
Name
School
Total
Mark/50

24th April 2015


Time Allowed: One hour

Attempt as many questions as you can.

Write your answers on this question paper.

Marks allocated for each question are shown in brackets on the right.

You may use any calculator.

You may use any standard formula sheet.

This is the first competition paper of the British Olympiad in Astronomy and Astrophysics.
To solve some of the questions, you will need to write equations, draw diagrams and, in general,
show your working.
This paper has real problems and is not like an A level paper. The questions are more difficult
because you are not told how to proceed. If you cannot do many, do not be disheartened. If
you can do some then you should be delighted. A good mark is from a few questions solved.
There are two optional parts that you may attempt after the exam. These are more difficult
questions that follow up on the questions to indicate how much information can be deduced from
the data by a keen astrophysicist.
Useful constants

Speed of light c 3.00 10 m s


Gravitational constant G 6.67 10 N m kg
Solar mass Msolar 1.99 10 kg
Astronomical Unit AU 1.496 10 m
Parsec pc 3.086 10 m
Earths orbit semi-major axis 1 AU
Earths rotation period 1 day 24 hours
Earths mass MEarth 5.97 10 kg
Earths axial tilt 23.4

You might find the diagram of an elliptical orbit below useful in solving some of the
questions:

P c a A
F O

Elements of an elliptic orbit:


semi-major axis
semi-minor axis
= ! eccentricity, where " = #$
F Sun/Earth - focus
P perihelion/perigee (point nearest to F)
A aphelion/apogee (point furthest from F)

Keplers Third Law:

For an elliptical orbit, the square of the period of orbit of a planet about the Sun is
proportional to the cube of the semi-major axis (a) (the average of the minimum and
maximum distances from the Sun).

List of symbols used in the paper:


- geographic latitude
L geographic longitude
UT Universal Time

1
Section A: Multiple Choice

Circle the correct answer to each question. Each question is worth 2 marks. There is only one
correct answer to each question. Total: 20 marks.

1. Why is the Moon heavily cratered, but not the Earth?


A. The Moon has stronger gravity, so it attracts more space debris
B. The Moon formed earlier than the Earth, so it had more time to be bombarded
by asteroids
C. The craters on Earth were eroded by the oceans and atmosphere over a long
period of time
D. The Moon orbits around the Earth in addition to orbiting around the Sun, so it
collects more space debris
2. We do not expect to find life on planets orbiting around high-mass stars because:
A. High-mass stars are far too luminous
B. The lifetime of a high-mass star is too short
C. High-mass stars are too hot to allow for life to form
D. Planets cannot have stable orbits around high-mass stars
3. What would happen to the Earths orbit if the Sun suddenly became a black hole with the
same mass?
A. It would spiral inwards because of the strong gravitational forces
B. It would fall on a straight line into the black hole
C. It would become an open orbit and the Earth would escape from the Solar
System
D. Nothing
4. A 10-inch refracting telescope with focal ratio (defined as the ratio of the focal length and
aperture) of 10 is used with a 25 mm focal length eyepiece. What is the magnifying power of
the telescope? %1 inch = 2.54 cm*
A. 10x
B. 50x
C. 100x
D. 200x
5. Which of the following planets has the longest day, defined as the period of a complete
rotation about its axis?

A. Venus B. Earth C. Mars D. Jupiter

2
6. Which of the following is not a zodiacal constellation?

A. Virgo B. Cancer C. Aquila D. Gemini


7. The Sun is seen setting from London (+ = 5130, N, . = 08 W* at 21:00 UT. At what
time UT will it be seen setting in Cardiff (+ = 5130, N, . = 311 W* on the same day?
A. 21:12
B. 21:00
C. 20:48
D. 20:58
8. How far away must your friend be standing from you such that the attractive force exerted
on you is similar to the maximum gravitational force exerted on you by Mars? Assume that
your friends mass is 65 kg. The mass of Mars is 6.4 x 1023 kg and the minimum distance
from Earth to Mars is 0.52 AU.
A. 2.3 m
B. 0.8 mm
C. 0.8 m
D. 2.3 mm
9. A comet follows an elliptical orbit that is 31.5 AU at aphelion and 0.5 AU at perihelion.
What is the period of the comet?
A. 181 years
B. 16 years
C. 64 years
D. 6.3 years
10. In which of the following places is the length of the shortest day of the year equal to half
the length of the longest night?
A. Dubai (+ = 25N*
B. London (+ = 52N*
C. Rio de Janeiro (+ = 23S*
D. Troms (+ = 70N*
[HINT: Do not attempt to calculate the latitude, but rather look at the answers and consider
how the length of the day varies with latitude and time of year.]

/20

3
Section B: Short Answer
Write your answers to the following questions. Each question is worth 5 marks. You should
show your working in the spaces provided. Total: 10 marks.

Question 11
A geostationary satellite is one that orbits in the equatorial plane of the Earth with the same
period and in the same direction as the Earths rotation. These orbits are important for
communication and weather observation because the satellite always remains above the same
point on Earth. The orbits of geostationary satellites are circular.

a) Calculate the radius of the orbit of a geostationary satellite. Ans: [3]

b) Imagine now that the satellite was orbiting the Earth at the same orbital radius and
same period, but in the opposite direction. For approximately how many hours a day
would a satellite be above the horizon for an observer at ground level, situated on the
Equator? Assume that the radius of the Earth can be neglected. Ans: [2]

/5

4
Question 12

The light from distant galaxies has distinct spectral features characteristic of the gas which
makes them up. The astronomer Edwin Hubble noticed that the lines in the spectra of most
galaxies are shifted towards the red end of the spectrum. This lead to his famous discovery
that the recessional velocity of a galaxy is proportional to the distance to the galaxy, the
constant of proportionality being 2 , implying that the Universe is expanding.

To measure the redshift of a galaxy, astronomers usually use the 3 parameter. Suppose that
we observe a galaxy with a redshift of 3 = 0.30 and find that one of the lines in the hydrogen
spectrum has been redshifted, compared to its rest wavelength of 486.1 nm. Assume that the
Universe is undergoing a uniform expansion, with the rate given by the Hubble constant,
2 = 72 km s Mpc .

8
a) Assuming that the classical Doppler effect 63 9 is a reasonable approximation,
what is the redshifted wavelength of the receding galaxy, in nm?
Ans: [3]

b) When we observe the galaxy, how far into its past are we looking? Ans: [2]

[A Mpc (abbreviation for megaparsec) is one million parsecs. It is a useful unit used by
astronomers to measure the large distances to galaxies]
/5

5
Section C: Long Answer
Write complete answers to the following questions. Total: 20 marks.

Question 13 Solar Eclipse


A major astronomical event happened on the morning of Friday 20th March 2015: a partial
solar eclipse visible from the whole of the UK (at least from the parts not fully covered by
clouds). The next partial solar eclipse of the same totality will happen in 2026 and the next
total solar eclipse visible from the UK will be in 2090. In the image below you can see a time
lapse of the eclipse, as seen from Sheffield, UK.

Figure 1 The partial solar eclipse visibility as seen from Sheffield, UK.

a) From the images in Figure 1 identify the time corresponding to the maximum of the
partial solar eclipse.

[1]

b) The apparent magnitude of an object is a measure of its brightness as seen by an


observer on Earth. Note that the brighter the object appears, the lower its magnitude.
From Figure 1, the maximum coverage of the solar eclipse, as seen from Sheffield,
was 90%. Using the relation between the difference in apparent magnitudes : and the
variation in brightness, #, also known as Pogsons formula, # # 2.512 %;< ;= * ,
estimate the magnitude of the Sun at the maximum of the eclipse, if the apparent
magnitude of the Sun is -26.74. Assume that the brightness of the solar disc is
uniform, therefore being proportional to the surface area.

6
[3]

c) The only two populated places where the totality could be seen were the Faroe and
Svalbard Islands. Explain if it would ever be possible to see a total solar eclipse from
the Capital of Svalbard, Longyearbyen (+ = 7813, N 1533, E* during December.

[1]

d) The tidal interaction between the Earth and the Moon causes the Moon to move away
from the Earth (increase its semi-major axis) by 3.82 cm/year, and the Earth to spin
down very slowly. Considering the most favourable case and using the data below
estimate in how many years a total solar eclipse will not be visible from anywhere on
Earth. Assume that the eccentricity of the Moons orbit does not change.
The radius of the Moon is @A = 1737.5 km, the mean distance to the Moon is A =
385,000 km and the eccentricity of the Moons orbit is A = 0.055. The radius of the Sun is
@B = 695,800 km, the mean distance to the Sun is C = 149.6 10 km and the
eccentricity of Earths orbit is C = 0.0167.
[HINT: Make use of the ellipse on page 1 and identify where the most favourable case lies,
considering the angular diameters of the Sun and of the Moon, respectively]

7
[7]

/12

Question 14 Transiting extrasolar planet


One method of detecting extrasolar planets is to observe their transit across the disc of their
host star. During the transit, the observed brightness of the star drops by a small amount,
depending on the size of the planet. In 1999, following the spectroscopic detection of a planet
around star HD 209458, astronomers David Charbonneau and Gregory Henry were able to
observe a transit of the planet across the disc of the star, making it the first detection of a
transiting extrasolar planet. The planet, named HD 209458b was found to be orbiting the star
with a mass of 1.15 Msolar on a circular orbit every 3.525 days, much faster than the Earth is
orbiting the Sun. Hundreds of extrasolar planets have since been detected using the transit
method by the Kepler mission. However, the main disadvantage of this method is that the
orbit of the planet has to be very close to edge-on, for the transit to occur from our vantage
point. In this question, assume that the planets orbit is perfectly edge-on, such that the transit
is central. The figures below are the plot of the light curve of star HD 209458, showing the
drop in brightness during the transit, and a schematic of the transit. Because the surface
brightness of the stars disc is not uniform (an effect called limb darkening), the real light
curve in Figure 2 does not fully resemble the idealised case in Figure 3.

Figure 2. The light curve of the star HD 209458. Figure 3. Schematic of the transit.

a) From Figure 2, what percentage of the stars disc is covered by the planet in the
middle of the transit? Estimate an error in your determination.

[1]

8
b) From your answer in a) determine the ratio of the radius of the planet and the radius of
the star.

[2]

c) Estimate the radius of the planets orbit in AU, assuming that the mass of the planet is
much smaller than the mass of the star.

[2]

d) From Figure 2 estimate the total transit time, from first to last contact (as shown in the
Figure 3). Assuming that the speed at which the transit occurs is equal to the circular
speed of the planet around the star, calculate the radii of the star and of the planet.
Express them in units of solar radii and Jupiter radii, respectively (@DEF = 6.96
10G km, @HEIJKLM = 7.0 10 km*.

____[3]

/8
END OF PAPER

9
British Olympiad in Astronomy and Astrophysics
April 2015
Solutions and marking guidelines for the BOAA Competition Paper

The total mark for each question is in bold on the right hand side of the table. The breakdown of the mark is below it.
There are multiple ways to solve some of the questions, so please accept all the good solutions that arrive at the correct
answer.

Question Answer Mark


Section A 20
1. C 2
2. B 2
3. D 2
4. C 2
5. A 2
6. C 2
7. A 2
8. C 2
9. C 2
10. B 2
Section B 10
11. a. Answer: 42,200 km 3
On a circular orbit, the centripetal force is due to the gravitational force:
=

= 1

=
4
1

= 24 hours, therefore the radius of the geostationary orbit is:

= 4.22 10 m 42,200 km
1

b. Answer: 6 hours 2

In the case when the satellite is orbiting the Earth with a period of # = 24 hours, but in
the opposite direction to Earths rotation, the relative angular velocity is:

4
=2 # = 1
#

The observer can be considered as being stationary on Earth and the satellite revolving
with . Because the radius of the geostationary orbit is larger than the radius of the Earth,

covering an angle of 180 (or ). The time the satellite is visible for the observer is:
the satellite will be visible above the horizon for the observer for half of its (relative) orbit,

$% = = = 6 hours
#
4 1
[Since the radius of the orbit is only 6 . , the visibility of the satellite is less than 6 hours.
In fact it is 5.42 hours.]
This occurs twice in a 24 hour period, so strictly it is 12 hours (or 10.8 h).
Either answer of 6 h or 12 h gains the mark.

1
12. a. Answer: 630 nm 3

From the classical Doppler effect: 3 0.3. This is a reasonable approximation as


long as 1 2, otherwise the full relativistic Doppler effect has to be used.

/ /# 1
Doppler shift formula:
= 3
/# 2
1

Where /# = 486.1 nm the rest wavelength. Hence:

/ = /# (1 + 3) 1
The observed (redshifted) wavelength is:

/ = 631.9 nm 630 nm 1

b. Answer: 4 billion years 2


The time it takes the light (travelling at speed 2) from the galaxy to reach us is:

$% =
1
2

According to Hubbles law the distance to the galaxy is:


1
=
:#

3 0.3
Hence,
$% ~ 4 billion years
:# 72 km s <= Mpc <= 1

In the equation above notice that the unit of :# is I <= and take care when doing the
conversion from Mpc to km.
Section C 20
13. a. 1
From Figure 1 identify that the maximum of the eclipse occurs between 09:24 and 09:34,
therefore in the image taken at 09:31.
b. 3
magnitude of the Sun is = = 26.74 corresponding to a brightness of = . During the
The brightness of the solar disc is proportional to the visible surface area. The apparent

eclipse, the Moon covers 90% of the solar disc, thus the visible area of the Sun is only
10%. The brightness of the solar disc during the maximum of the eclipse, is:

= 0.1 = 1

This corresponds to an apparent magnitude . Inverting the Pogsons formula, the


apparent magnitude of the Sun during the eclipse is:

log=#
= =

=
log=# 2.512

= = 2.5 log=# 1
=

= 26.74 2.5 log=# 0.1

= 24.24 1

2
The Sun was 2.5 magnitudes less bright during the eclipse. It appeared to be slightly
dimmer outside, indicating that only 10% of the Sun was able to provide sufficient light to
continue our daily activities. Even with a small percentage of the Sun being visible, it is
still 40,000 brighter than the full Moon, whose apparent magnitude is -12.74.
c.
Longyearbyen (latitude 7813L N) is situated above the Arctic circle (latitude 6634L N =
1
90 2327) and it experiences polar night during the whole month of December (from
November to February, more specifically). Hence, the Sun is not visible above the horizon
during these months and a solar eclipse would not be observable. Luckily, the total solar
eclipse occurred in March.
d. 7
Total solar eclipses would no longer be visible from Earth when the angular size of the
Moon will be smaller than the angular size of the Sun. To calculate the angular sizes of the
Sun and of the Moon, respectively, make use of a diagram such as the one below:


R
d

From the diagram, the apparent diameter of the object is:

R
O = 2 sin<= P S
d
1

For small angles, sin O tan O O (in radians), so any of these is acceptable.

The most favourable case for the total eclipse not to occur is for the Moon to have the
largest possible angular size, and the Sun the smallest possible angular size. This happens 1
when the Moon is at perigee (nearest point to the Earth) and the Earth at aphelion (furthest
point from the Sun). First calculate the distance to the Sun, T U and its angular diameter.

From the diagram of the ellipse on page 1, the distance to the aphelion is:

T U = V + 2 = V (1 + W ) = 152.1 10X km 1
The angular diameter of the Sun (and of the Moon for total eclipses not to occur) is:

O = 0.524 1
Now calculate where the Moon must be, YT[ and hence obtain VZ . Then calculate the
change of VZ from current value and, knowing the rate, calculate the time taken.
The distance the Moon needs to be situated at is:

.Z
Y = O 380,000 km
1
sin
2

This will be the perigee of the new orbit, YT[ . Again, from the diagram of the ellipse on
page 1, the distance to the perigee is:

YT[ = V 2 = VZ (1 WZ )

Hence, the new semi-major axis of the Moon will be:

YT[
VZ = 402,120 km
1
1 WZ

At a rate of 3.82 cm/year, the Moon will move from the current semi-major axis of

$% 450 million years


385,000 km to 402,120 km in:
1

In 450 million years we will only be able to observe annular solar eclipses from Earth.

3
14 a. 1
The percentage of the stars disc covered by the planet is 1.65%. 0.5
The light-curve has high quality data, with an error of only ~ 0.05%
0.5
[Accept 1.6 0.1% or 1.7 0.1% and the errors propagated in the following
calculations]
b. 2
The covered area of the star, during the transit is:

`ab = .ab

The percentage determined in a) of c = 0.0165 corresponds to:

`ab .ab
c = =
`d .d 1
Therefore, the ratio of the radius of the planet and of the star is:

.ab
= ec = 0.128
.d 1
c. 2
4
Use Newtons third law (which gives the proportionality of Kepler III but with constants):
=
V f ( hijk + lmjnoi )

Neglect mlmjnoi Mhijk and find the radius of the planets orbit:

V =
d
4
1

Using the values in the question, Mhijk = 1.15 Mhrmjk and the period of T = 3.525 days
radius of the planets orbit is:

V = 0.047 AU 1

d. 3
From the light curve (Figure 2), the total transit time (from the beginning of the drop, to its
end) is:

$% t b 0.13 days 3.12 hours 1

The transit time is equivalent to the duration of an eclipse from first to last contact, as seen

2(.d + .ab ). Because the star and the planet are practically at the distance from the
in Figure 3. During the eclipse, the centre of the planets disc moves a distance equal to

around the star, 1ab :


observer, the speed at which the transit occurs is equal to the circular speed of the planet

2(.d + .ab ) 2 V
1ab = =
1
$% t b

$%
Hence,
.d + .ab = V ~ 8.15 10u km
t b

And using the ratio in b), 0.128, the radius of the star and of the planet are:

.d = 7.23 10u km 1.04 .vw 0.5


.ab = 0.92 10u km 1.32 .xwa[
0.5

4
British Olympiad in Astronomy and Astrophysics

Trial Paper
Name
School

Total
March 2015 Mark/50
Time Allowed: One hour

Attempt as many questions as you can.

Write your answers on this question paper.

Marks allocated for each question are shown in brackets on the right.

You may use any calculator.

You may use any standard formula sheet.

This is a trial paper for the British Olympiad in Astronomy and Astrophysics. The first
competition paper of the British Olympiad in Astronomy and Astrophysics will take place in
April 2015 and will have a similar format and questions to this trial paper.

To solve some of the questions, you will need to write equations, draw diagrams and, in general,
show your working. There are two optional parts that you should attempt in extra time. These
are more difficult questions that will not be marked, but they are useful for your training.
Useful constants

Speed of light c 3.00 10 m s


Gravitational constant G 6.67 10 N m kg
Solar mass Msolar 1.99 10 kg
Astronomical Unit AU 1.496 10 m
Earths orbit semi-major axis 1 AU
Earths orbital period 1 year 365.25 days

1
Section A: Multiple Choice

Circle the correct answer to each question. Each question is worth 2 marks. There is only one
correct answer to each question. Total: 20 marks.

1. Which of the following types of stars is the hottest?


A. Red giant
B. Brown dwarf
C. O-type blue giant
D. Yellow main sequence star
2. The majority of the mass in the Universe is contained in:
A. The most massive stars
B. Gas and dust
C. Dark matter
D. Supermassive black holes
3. How much more or less light can an 8-metre aperture telescope collect compared to a 4-
metre aperture telescope (in the same amount of time)?
A. Half the amount
B. The same amount
C. Twice as much
D. Four times as much
4. Why arent solar/lunar eclipses observed at every new and full moon?
A. Because the orbital plane of the Moon is tilted compared to the Earths orbit
around the Sun
B. Eclipses happen every month, but they can be seen from different places on
Earth
C. Most eclipses happen during the day, so they are not visible
D. Because the Earth-Moon distance changes in time
5. Which planet would you not be able to see on the night sky at midnight from the UK?

A. Jupiter B. Venus C. Mars D. Saturn

2
6. Which of the following constellations is not visible from the UK?

A. Canis Major B. Cygnus C. Crux D. Gemini

7. What is Earths mean orbital speed around the Sun?


A. 150 km s
B. 30 km s
C. 15 km s
D. 0.3 km s
8. If your mass is 65 kg, what is the maximum value of the attractive force exerted on you by
Jupiter? The mass of Jupiter is 1.9 x 1027 kg and its semi-major axis is 5.2 AU.
A. 1.4 x 10-5 N
B. 2.1 x 10-11 N
C. 9.6 x 10-6 N
D. 2.1 x 10-5 N
9. The famous comet, Halleys comet, appeared in the night sky in 1986. The semi-major axis
of its orbit is 17.8 AU. When is it going to return next?
A. 2023
B. 2061
C. 2064
D. 2093
10. Suppose a colony is established on Mars. How long would it take for a Martian doctor to
send a question to a colleague on Earth and receive a response, when Mars is closest to Earth?
Assume that the colleague replies instantly. The radius of Marss orbit is 1.524 AU.
A. 8.7 minutes
B. 25.3 minutes
C. 4.3 minutes
D. 12.7 minutes

/20

3
Section B: Short Answer
Write your answers to the following questions. Each question is worth 5 marks. You should
show your working in the spaces provided. Total: 10 marks.

Question 11
In the image below you can see the projection of the shadow of a child onto the wall he faces.
The height of the child from head to toes is 1.8 m, the length of the shadow on the wall is 0.8
m and on the ground is 1.6 m.

[HINT: It is useful to draw a diagram of the child, the wall and the position of the Sun]

a) What is the altitude of the Sun above the horizon? Ans: [3]

b) What would be the length of the shadow in the absence of the wall? Ans: [2]

/5

4
Question 12

An astronomer observes a galaxy and finds that one of the lines in the hydrogen spectrum has
been redshifted to 669.4 nm, compared to the rest wavelength of 656.3 nm. Assume that the
Universe is undergoing a uniform expansion, with the rate given by the Hubble constant,
= 72 km s Mpc .

a) The shift of the line is due to the Doppler effect. What is the velocity v of the receding
galaxy, in km s ? Ans: [3]

b) Edwin Hubble discovered his famous law that the recession velocity of a galaxy is
proportional to the distance to the galaxy, the constant of proportionality being .
What is the distance r to the galaxy in Mpc? Ans: [2]

[A Mpc (abbreviation for megaparsec) is one million parsecs. It is a useful unit used by
astronomers to measure the large distances to galaxies]

/5

5
Section C: Long Answer
Write complete answers to the following questions. Each question is worth 10 marks. The last
point of each question is optional for your training only. Total: 20 marks.

Question 13 Solar Eclipse

On the morning of Friday 20th March 2015 a partial solar eclipse will be visible from the
whole of the UK. Solar eclipses are quite rare and this will be a major event, with the Moon
passing in front of the Sun and covering a large portion of the solar disc. This will be an event
you will remember for the rest of your life, but remember you shouldnt watch the Sun
without a suitable filter!

The radius of the Moon is 1737.5 km and the distance to the Moon will be 365,100 km on that
day. The radius of the Sun is 695,800 km and the distance to the Sun is 149.6 million km. The
Moon orbits the Earth, in an anticlockwise direction (viewed from the above the North Pole),
the same direction as the Earth rotates about its axis. The period of the Moons orbit around
the Earth, relative to the Sun (the synodic period the period between when the Sun, Moon
and Earth are in line), is 29.5 days. Using this information:

a) Calculate the angular diameters in degrees (how large they appear) of the Sun and of
the Moon, respectively, as they will be seen in the sky on that day.

[3]

b) To observe the eclipse you will be using a telescope with a focal length of 200 cm and
an eyepiece with a focal length of 25 mm and field of view (FOV) of 52. Is it
possible to see the entire image of the solar disc in the eyepiece of this telescope?

[2]

6
c) Suppose that you are observing the eclipse from a place near the North Pole where the
Earths spin can be neglected. Calculate the duration of the eclipse from the first to
last contact, assuming that the eclipse is central.

[3]

d) Explain if the image below (Figure 1) shows the beginning or end of the solar eclipse.

Figure 1. Binocular view of the solar eclipse. The N and E directions for the observer are shown.

[2]

/10

7
e) Optional

Estimate the duration of the eclipse from the first to last contact, assuming that the eclipse is
central, for an observer situated in London (latitude = 52.5). In this case, Earths spin
cannot be neglected. The radius of the Earth is !"#$% = 6370 km and the spin period is
&!"#$% = 24 hours. (the distance to the Moon can be taken to be the same).

8
Question 14 Extrasolar planet discovery
In 1995 a team of Swiss astronomers from the Geneva Observatory announced that they had
discovered the first planet outside our solar system around the star 51 Pegasi. They found it
by looking at the spectrum of the star and observing the slight change in its velocity, as the
star and planet move around their common centre of mass. The planet was found to be
orbiting the star on a near circular orbit every 4.23 days, much faster than the Earth is orbiting
the Sun. Since then, the radial velocity method that relies on the Doppler effect has been used
to discover hundreds of extrasolar planets. The figure below is the original plot of the radial
velocity of star 51 Pegasi as it varies with the time.

A C

Figure 2. The radial velocity curve of the star 51 Pegasi. The


phase of 1 is equivalent to one full period of the planet around
the star.

a) In the figure below (Figure 3), mark in the boxes the letters (A, B, C and D)
corresponding to the positions of the star 51 Pegasi around the star-planet centre of
mass, as inferred from the radial velocities in Figure 1. Assume that the star is orbiting
in a clockwise direction. The figure is not drawn to scale.

Direction to observer Centre of mass

Figure 3.

[2]

9
b) From Figure 2, determine the velocity of the star around the star-planet centre of mass
and estimate an error in your determination.

[1]

c) The star 51 Pegasi has a similar mass to the Sun. Estimate the distance from the planet
to the star in AU, assuming that the mass of the planet is much less than that of the
star.

[3]

d) Estimate the mass of the planet and express it in terms of Jupiter masses, +,-./$0# =
1.9 10 1 kg. Why is this a lower limit for the real mass of the planet?

[HINT: For a binary system 2 3 = 2 3 , where 3 is the distance from the object with mass
2 to the centre of mass of the system]

[4]

/10

10
e) Optional

Without making the approximation that the mass of the planet is much less than the mass of
51 Pegasi, calculate the mass of the planet (expressed in terms of Jupiter masses +,-./$0# =
456789:
1.9 10 1 kg) by obtaining an expression involving the ratio 4;:7<
.

END OF PAPER

11
Solutions for the BOAA trial paper

Question Answer Mark


Section A 20
1. C 2
2. C 2
3. D 2
4. A 2
5. B 2
6. C 2
7. B 2
8. D 2
9. B 2
10. A 2
Section B 10
11. a. 32 3
b. 2.9 m 2

Solution:

H
h

l
a. Let H = 1.8 m, h = 0.8 m, l = 1.6 m. From the information in the
question and the diagram above, the altitude of the Sun is:

tan 32

b. The length of the shadow in the absence of the wall is:

2.9 m
tan

12. a. 6.00 10 km s 3
b. 83 Mpc 2

Solution:

a. Doppler shift:

Where 656.3 nm the rest wavelength and 669.4 nm the


observed wavelength. Hence, 5988 km s ~ 6000 km s .

b. Hubble law:

Using v and the given H0, 83.17 Mpc ~ 83 Mpc .

1
Section C 20
13. a. 3
Diagram
R
1
d

From the diagram, the apparent diameter of the object is:


R
2 sin 1
d

For small angles, sin 1 2 tan 1 2 1 3in radians5, so any of these is


acceptable.
Numerically:
Sun: 0.533 1
Moon: 0.545

b. 2
The magnification of the telescope is:

789:;<=>?; 200 cm
6 80 1
7;@;A>;<; 2.5 cm

The field of view of the telescope is thus:

BCD;@;A>;<; 52
BCD=;E;F<8A; 0.65
1
6 80

The FOV is larger than the apparent diameter of the Sun (0.5335, so it is
possible to see the entire image of the Sun in the eyepiece.

c. 3
At the North Pole the Earth is static. The reason why the eclipse occurs is
that the Moon has its own motion around the Earth with a period of 29.5
days, relative to the Earths motion around the Sun. The angular velocity
of the Moon is thus:

2 360
G 12.2/day
I 29.5 days 1
Last contact First contact

As seen in the diagram above, the centre of the Moon covers an angular 1
distance of L OPQ R S88Q 1.078 from first to last contact.

The time needed for the Moon to cover this distance is:

L 1.078
LM days 0.088 days 2.12 hours
G 12.2 1
Thus, the duration of the eclipse seen from the North Pole is 2.12 hours.

2
d. 2
Earth rotates about its axis from West to East (anticlockwise direction), so
the Sun and Moon appear to move in the sky from East to West
(clockwise). The Moon orbits the Earth, in an anticlockwise direction,
from W to E, so the eclipse will begin on the W side of the Sun and will
end in the E (as seen by an observer on Earth). Judging from the
coordinates given in the image, this is the beginning of the solar eclipse.

e. Optional

In part (c) we calculated the duration of the eclipse in case of a static


Earth, LM 2.12 hours. This is a special case that only occurs at the Poles
of the Earth. At any other latitudes, we need to consider Earths spin in the
calculations, as the observer will be moving along Earths surface. This
will extend the duration of the eclipse, since the observer and the Moon
will rotate in the same direction (anticlockwise). During the eclipse, the
shadow of the Moon moves on the surface of the Earth with a linear
velocity:

2XYUVVW
UVVW 2 3,240 km h
IUVVW

The distance the shadow of the Moon travels on the static Earth (assuming
that the Earth had a flat surface) during the LM 2.12 hours of eclipse is:

UVVW LM 6,870 km

Now, consider the rotating Earth. Earths rotational velocity at the Equator
is:

2X`[\]^_
[\]^_ 2 1,670 km h
I[\]^_

At Londons latitude 3b 52.5), Earths rotational velocity is:

[\]^_,a [\]^_ cos b 2 1,020 km h

The Earth-Moon relative velocity is thus:

]cd UVVW [\]^_,a 2 2220 km h

In this case, the duration of the eclipse will increase to:

UVVW
LM e LM 2 3.1 hours
]cd UVVW [\]^_ cos b

In reality, the duration of the eclipse, as viewed from London, will be less
than 3.1 hours as we need to consider that the Earths surface is curved. So
far, we considered that the shadow of the Moon moves on a projection of
Earths curved surface on a flat surface, but an accurate calculation is
complicated.

Also, in the question we considered the case of a total eclipse (the eclipse
is central), while from London the eclipse on 20th March will be a partial
one, with an obscuration of 85%. Therefore, the angular distance the Moon
covers from first to last contact is smaller than the one we calculated.

The partial eclipse on 20th March will last for 2h16min, with first contact at
08:45 UT and last contact at 10:41 UT.

3
14 a. 2

0.5
marks
C A each

b. 1
From Figure 2 the velocity of the star around the centre of mass is:

f^\] 60 10 m s

Accepted values for the velocity between 58 60 m s 0.5


marks
Accepted values for the uncertainty in velocity between 10 14 m s each

c. 3
Use Keplers third law:
Ih 4h
1
Y i36f^\] R jkd\Wc^ 5

Neglect mkd\Wc^ Mf^\] , and because the mass of 51 Pegasi is the same as
of the Sun use:
Ih
1
Y
1

Where T is in years and a in AU. The period of the planet is I


4.23 days therefore the semi-major axis (same with radius in this case as
the orbit is nearly circular) of the planets orbit is:

Y 0.05 AU 1

d. 4
The position of the centre of mass, in the CM frame is zero. I.e.

jkd\Wc^ pqrstuv R 6f^\] pwvsx


o 0
jkd\Wc^ R 6f^\]
Therefore:
jkd\Wc^ kd\Wc^ 6f^\] f^\]

The star-CM distance:


2X f^\]
F^\]
I 1
From where we get:
6fVd\] F^\] I
jkd\Wc^
2X kd\Wc^ 1

jkd\Wc^ 9.3 10hy kg 2 0.5 6{|k}^c] 1

4
Different method
For a closed system (like this planet-star system) the total linear
momentum, in the centre of mass frame, is 0. Therefore, the momenta of
the star and of the planet are equal.

Conservation of momentum:
jkd\Wc^ kd\Wc^ 6f^\] F^\]

Circular velocity:
2X kd\Wc^
kd\Wc^
I

6f^\~ 6fVd\]

Mass of the planet is:


6fVd\] f^\] I
jkd\Wc^ 2 0.5 6{|k}^c]
2 kd\Wc^

This is a lower estimate of the mass of the planet because we are not given
any information about the inclination of the orbit, which we assume to be
edge-on. If the orbit were tilted by an angle i to the line of sight, the 1
measured radial velocity would be v sin i, and hence the true mass of the
planet is: 6=~P; 6>Q . Unfortunately, using the radial velocity
method we are not able to determine the inclination of the system, so all
the masses we measure are lower estimates.
[Any explanation about the tilt of the orbit is acceptable]
e. Optional

In part (c) we neglected the mass of the planet as jkd\Wc^ 6f^\] .


If we dont neglect it Keplers 3rd law becomes:

I h i 6f^\] R jkd\Wc^
R f^\]
4h kd\Wc^

Using the hint in the question:


F
jA 6F
A

Replacing in Keplers 3rd law:

I h i 6F A R F
A R F
4h A
Rearranging,

I h i 6F A A
h
1R
4h F F F

From Figure 3 we can get the radius of the orbit of 51 Pegasi:

f^\] I
F 3.49 10y m
2

Replacing numerically,
1.057 10 131 R 15h where 1 A
F

Hence 1 2195 and


jkd\Wc^ 0.48 6{|k}^c]

Vous aimerez peut-être aussi